Pharm II Practice Questions

अब Quizwiz के साथ अपने होमवर्क और परीक्षाओं को एस करें!

5 Before initiating therapy with a nonselective beta blocker, the nurse will assess the patient for a history of which condition? a Hypertension b Liver disease c Pancreatitis d Asthma

5. d

5 While monitoring a patient who is receiving an infusion of a crystalloid solution, the nurse will monitor for which potential problem? a Bradycardia b Hypotension c Decreased skin turgor d Fluid overload

5. d

Which nursing diagnosis is appropriate for a patient who has received a sedative-hypnotic drug? A. Ineffective peripheral tissue perfusion B. Fluid volume excess C. Risk for injury D. Risk for infection

C. Risk for injury Rationale Sedative-hypnotics cause central nervous system (CNS) depression, putting the patient at risk for injury.

1 A patient who has a history of asthma is experiencing an acute episode of shortness of breath and needs to take a medication for immediate relief. The nurse will choose which medication that is appropriate for this situation? a A beta agonist, such as albuterol b An leukotriene receptor antagonist, such as montelukast c A corticosteroid, such as fluticasone d An anticholinergic, such as ipratropium

1. a

1 The nurse caring for a patient who is receiving beta1 agonist drug therapy needs to be aware that these drugs cause which effect? a Increased cardiac contractility b Decreased heart rate c Bronchoconstriction d Increased GI tract motility

1. a

1 The nurse is providing education about cholinergic-blocking drug therapy to an elderly patient. Which is an important point to emphasize for this patient? a Avoid exposure to high temperatures. b Limit liquid intake to avoid fluid overload. c Begin an exercise program to avoid adverse effects. d Stop the medication if excessive mouth dryness occurs.

1. a

1 When assessing a patient who is to receive a decongestant, the nurse will recognize that a potential contraindication to this drug would be which condition? a Glaucoma b Fever c Peptic ulcer disease d Allergic rhinitis

1. a

1 Which action by the nurse is most appropriate for the patient receiving an infusion of packed red blood cells? a Flush the IV line with normal saline before the blood is added to the infusion. b Flush the IV line with dextrose before the blood is added to the infusion. c Check the patient's vital signs once the infusion is completed. d Anticipate that flushed skin and fever are expected reactions to a blood transfusion.

1. a

1 A patient has been admitted to the emergency department because of an overdose of an oral benzodiazepine. He is very drowsy but still responsive. The nurse will prepare for which immediate intervention? a Hemodialysis to remove the medication b Administration of flumazenil c Administration of naloxone d Intubation and mechanical ventilation

1. b

1 A patient with narcolepsy will begin treatment with a CNS stimulant. The nurse expects to see which adverse effect? a Bradycardia b Nervousness c Mental clouding d Drowsiness at night

1. b

1 The nurse is reviewing the use of bethanechol (Urecholine) in a patient who is experiencing postoperative urinary retention. Which statement best describes the mechanism of action of bethanechol? a It causes decreased bladder tone and motility. b It causes increased bladder tone and motility. c It increases the sensation of a full bladder. d It causes the sphincters in the bladder to become tighter.

1. b

1 When a patient has experienced extravasation of a peripheral infusion of dopamine, the nurse will inject the alpha blocker phentolamine (Regitine) into the area of extravasation and expect which effect? a Vasoconstriction b Vasodilation c Analgesia d Hypotension

1. b

1 When assessing a patient who will be receiving a measles vaccine, the nurse will consider which condition to be a possible contraindication? a Anemia b Pregnancy c Ear infection d Common cold

1. b

1 When monitoring for a therapeutic response to aminoglutethimide, the nurse will look for which potential outcomes? a Increase in Cushing's syndrome characteristics b Decrease in Cushing's syndrome characteristics c Increased lymphocyte levels d Growth suppression

1. b

In caring for a patient experiencing ethanol withdrawal, the nurse expects to administer which medication or medication class as treatment for this condition? a lithium (Eskalith) b Benzodiazepines c buspirone (BuSpar) d Antidepressants

1. b

1 The nurse is assessing a patient who is to receive dinoprostone (Prostin E2). Which condition would be a contraindication to the use of this drug? a Pregnancy at 15 weeks' gestation b GI upset or ulcer disease c Ectopic pregnancy d Incomplete abortion

1. c

1 A 30-year-old man is taking simethicone for excessive flatus associated with diverticulitis. During a patient teaching session, the nurse explains the mechanism of action of simethicone by saying: a "It neutralizes gastric pH, thereby preventing gas." b "It buffers the effects of pepsin on the gastric wall." c "It decreases gastric acid secretion and thereby minimizes flatus." d "It causes mucus-coated gas bubbles to break into smaller ones."

1. d

1 A patient has a new order for glatiramer acetate. The patient has not had an organ transplant. The nurse knows that the patient is receiving this drug for which condition? a Psoriasis b Rheumatoid arthritis c Irritable bowel syndrome d Relapse-remitting multiple sclerosis

1. d

1 During treatment with zidovudine, the nurse needs to monitor for which potential adverse effect? a Retinitis b Deep vein thromboses c Kaposi's sarcoma d Bone marrow suppression

1. d

2 After a nebulizer treatment with the beta agonist albuterol, the patient complains of feeling a little "shaky," with slight tremors of the hands. The patient's heart rate is 98 beats/min, increased from the pretreatment rate of 88 beats/min. The nurse knows that this reaction is an a expected adverse effect of the medication. b allergic reaction to the medication. c indication that he has received an overdose of the medication. d idiosyncratic reaction to the medication.

2. a

2 During a teaching session for a patient who is receiving inhaled salmeterol, the nurse emphasizes that the drug is indicated for which condition? a Rescue treatment of acute bronchospasms b Prevention of bronchospasms c Reduction of airway inflammation d Long-term treatment of sinus congestion

2. b

2 When teaching a patient who is taking oral contraceptive therapy for the first time, the nurse relates that adverse effects may include which of the following? a Dizziness b Nausea c Tingling in the extremities d Polyuria

2. b

2 While assessing a patient who is to receive muromonab-CD3, the nurse knows that which condition would be a contraindication for this drug? a Acute myalgia b Fluid overload c Polycythemia d Diabetes mellitus

2. b

2 After giving an injection to a patient with HIV infection, the nurse accidentally receives a needlestick from a too-full needle disposal box. Recommendations for occupational HIV exposure may include the use of which drug(s)? a didanosine b lamivudine and enfuvirtide c zidovudine, lamivudine, and indinavir d acyclovir

2. c

2 An older adult has been given a benzodiazepine for sleep induction, but the night nurse noted that the patient was awake most of the night, watching television and reading in bed. The nurse documents that the patient has had which type of reaction to the medication? a Allergic b Teratogenic c Paradoxical d Idiopathic

2. c

2 The family of a patient who has recently been diagnosed with Alzheimer's disease is asking about the new drug prescribed to treat this disease. The patient's wife says, "I'm so excited that there are drugs that can cure this disease! I can't wait for him to start treatment." Which reply from the nurse is appropriate? a "The sooner he starts the medicine, the sooner it can have this effect." b "These effects won't be seen for a few months." c "These drugs do not cure Alzheimer's disease. Let's talk about what the physician said to expect with this drug therapy." d "His response to this drug therapy will depend on how far along he is in the disease process."

2. c

2 The nurse has provided teaching about oral corticosteroid therapy to a patient. Which statement by the patient shows a need for more teaching? a "I will report any fever or sore throat symptoms." b "I will stay away from anyone who has a cold or infection." c "I can stop this medication if I have severe adverse effects." d "I will take this drug with food or milk."

2. c

2 When administering beta blockers, the nurse will follow which guideline for administration and monitoring? a The drug may be discontinued at any time. b Postural hypotension rarely occurs with this drug. c Tapering off the medication is necessary to prevent rebound hypertension. d The patient needs to stop taking the medication at once if he or she gains 3 to 4 pounds in a week.

2. c

2 When evaluating the medication list of a patient who will be starting therapy with an H2 receptor antagonist, the nurse is aware that which drug may interact with it? a codeine b penicillin c ketoconazole d acetaminophen

2. c

2 When giving a vaccination to an infant, the nurse should tell the mother to expect which adverse effect? a Fever over 101° F (38.3° C) b Rash c Soreness at the injection site d Chills

2. c

2 When giving decongestants, the nurse must remember that these drugs have alpha-adrenergic-stimulating effects that may result in which effect? a Fever b Bradycardia c Hypertension d CNS depression

2. c

2 When preparing an IV solution that contains potassium, the nurse knows that a contraindication to the potassium infusion would be a diarrhea. b serum sodium level of 145 mEq/L. c serum potassium level of 5.6 mEq/L. d dehydration.

2. c

2 A patient at a weight management clinic who was given a prescription for orlistat (Xenical) calls the clinic hotline complaining of a "terrible side effect." The nurse suspects that the patient is referring to which problem? a Nausea b Sexual dysfunction c Urinary incontinence d Fecal incontinence

2. d

2 Patient teaching for a patient receiving an MAOI would include instructions to the patient to avoid which food product? a Orange juice b Milk c Shrimp d Swiss cheese

2. d

2 The nurse is giving a cholinergic-blocking drug and will assess the patient for which contraindications to these drugs? a Chronic bronchitis b Peptic ulcer disease c Irritable bowel syndrome d Benign prostatic hyperplasia

2. d

3 In the emergency department, several patients have possibly been exposed to anthrax. The nurse will prepare to administer prophylactic doses of a ciprofloxacin. b cidofovir. c immunoglobulin. d antitoxin.

3. a

3 When the nurse is teaching a patient who is taking acyclovir for genital herpes, which statement by the nurse is accurate? a "This drug will help the lesions to dry and crust over." b "Acyclovir will eradicate the herpes virus." c "This drug will prevent the spread of this virus to others." d "Be sure to give this drug to your partner, too."

3. a

3 A patient has been receiving an aminophylline (xanthine derivative) infusion for 24 hours. The nurse will assess for which adverse effect when assessing the patient during the infusion? a CNS depression b Sinus tachycardia c Increased appetite d Temporary urinary retention

3. b

3 During long-term corticosteroid therapy, the nurse will monitor the patient for Cushing's syndrome, which is manifested by a weight loss. b moon face. c hypotension. d thickened hair growth.

3. b

3 The nurse is giving a dose of bethanechol (Urecholine) to a postoperative patient. The nurse is aware that contraindications to bethanechol include: a bladder atony. b peptic ulcer. c urinary retention. d hypothyroidism.

3. b

3 The nurse is preparing to administer a medication for sleep. Which intervention applies to the administration of a nonbenzodiazepine, such as zaleplon (Sonata)? a These drugs need to be taken about 1 hour before bedtime. b Because of their rapid onset, these drugs need to be taken just before bedtime. c The patient needs to be cautioned about the high incidence of morning drowsiness that may occur after taking these drugs. d These drugs are less likely to interact with alcohol.

3. b

3 The nurse is reviewing a patient's medication orders for prn (as necessary) medications that can be given to a patient who has bronchitis with a productive cough. Which drug will the nurse choose? a An antitussive b An expectorant c An antihistamine d A decongestant

3. b

3 When administering sucralfate, which action by the nurse is most correct? a Giving the drug with meals b Giving the drug on an empty stomach c Instructing the patient to restrict fluids d Waiting 30 minutes before administering other drugs

3. b

3 When assessing a patient who is about to receive an albumin infusion, the nurse knows that a contraindication for albumin would be a acute liver failure. b heart failure. c severe burns. d fluid-volume deficit.

3. b

3 When assessing for adverse effects of cholinergic-blocking drug therapy, the nurse would expect to find that the patient complains of which drug effect? a Diaphoresis b Dry mouth c Diarrhea d Urinary frequency

3. b

3 After a patient has been treated for depression for 4 weeks, the nurse calls the patient to schedule a follow-up visit. What concern will the nurse assess for during the conversation with the patient? a Weakness b Hallucinations c Suicidal ideations d Difficulty with urination

3. c

3 For a patient receiving a vasoactive drug such as intravenous dopamine, which action by the nurse is most appropriate? a Monitor the gravity drip infusion closely, and adjust as needed. b Assess the patient's cardiac function by checking the radial pulse. c Assess the intravenous site hourly for possible infiltration. d Administer the drug by intravenous boluses according to the patient's blood pressure.

3. c

3 The nurse providing teaching for a patient who has a new prescription for beta1 blockers will keep in mind that these drugs may result in which effect? a Tachycardia b Tachypnea c Bradycardia d Bradypnea

3. c

3 During therapy with azathioprine (Imuran), the nurse must monitor for which common adverse effect? a Bradycardia b Diarrhea c Vomiting d Thrombocytopenia

3. d

3 The nurse is developing a plan of care for a patient receiving an anorexiant. Which nursing diagnosis is most appropriate? a Deficient fluid volume b Sleep deprivation c Impaired memory d Imbalanced nutrition, less than body requirements

3. d

3 The nurse is reviewing the use of obstetric drugs. Which situation is an indication for an oxytocin (Pitocin) infusion? a Termination of a pregnancy at 12 weeks b Hypertonic uterus c Cervical stenosis in a patient who is in labor d Induction of labor at full term

3. d

4 A patient took an accidental overdose of a cholinergic drug while at home. He comes to the emergency department with severe abdominal cramping and bloody diarrhea. The nurse expects that which drug will be used to treat this patient? a atropine (generic) b physostigmine (Antilirium) c bethanechol (Urecholine) d phentolamine (Regitine)

4. a

4 The nurse administering a cholinergic-blocking drug to a patient who is experiencing drug-induced extrapyramidal effects would assess for which therapeutic effect? a Decreased muscle rigidity and tremors b Increased heart rate c Decreased bronchial secretions d Decreased GI motility and peristalsis

4. a

4 The nurse has provided patient education regarding therapy with the SERM raloxifene (Evista). Which statement from the patient reflects a good understanding of the instruction? a "When I take that long flight to Asia, I will need to stop taking this drug at least 3 days before I travel." b "I can continue this drug even when traveling as long as I take it with a full glass of water each time." c "After I take this drug, I must sit upright for at least 30 minutes." d "One advantage of this drug is that it will reduce my hot flashes."

4. a

4 The nurse will monitor the patient who is taking a muscle relaxant for which adverse effect? a CNS depression b Hypertension c Peripheral edema d Blurred vision

4. a

4 When teaching a patient who has been prescribed a daily dose of prednisone (Deltasone), the nurse knows that the patient will be told to take the medication at which time of day to help reduce adrenal suppression? a In the morning b At lunchtime c At dinnertime d At bedtime

4. a

4 A patient has a new prescription for sumatriptan (Imitrex). The nurse providing patient teaching on self-administration will include which information? a Correct technique for intramuscular injections b Take the medication before the headache worsens. c Allow at least 30 minutes between injections. d Take no more than 4 doses in a 24-hour period.

4. b

4 A patient with a history of renal problems is asking for advice about which antacid he should use. The nurse will make which recommendation? a "Patients with renal problems cannot use antacids." b "Aluminum-based antacids are the best choice for you." c "Calcium-based antacids are the best choice for you." d "Magnesium-based antacids are the best choice for you."

4. b

4 During a routine checkup, a 72-year-old patient is advised to receive an influenza vaccine injection. He questions this, saying, "I had one last year. Why do I need another one?" What is an appropriate response from the nurse? a "The effectiveness of the vaccine wears off after 6 months." b "Each year a new vaccine is developed based on the flu strains that are likely to be in circulation." c "When you reach 65 years of age, you need boosters on an annual basis." d "Taking the flu vaccine each year allows you to build your immunity to a higher level each time."

4. b

4 The nurse is caring for a patient who has been taking clozapine (Clozaril) for 2 months. Which laboratory test(s) should be performed regularly while the patient is taking this medication? a Platelet count b WBC count c Liver function studies d Renal function studies

4. b

4 During a teaching session for a patient receiving an immunosuppressant drug, the nurse will include which statement? a "It is better to use oral forms of these drugs to prevent the occurrence of thrush." b "You will remain on antibiotics to prevent infections." c "It is important to use some form of contraception during treatment and for up to 12 weeks after the end of therapy." d "Be sure to take your medications with grapefruit juice to increase absorption."

4. c

4 During a teaching session for a patient who will be receiving a new prescription for the LTRA montelukast (Singulair), the nurse will tell the patient that the drug has which therapeutic effect? a Improves the respiratory drive b Loosens and removes thickened secretions c Reduces inflammation in the airway d Stimulates immediate bronchodilation

4. c

4 The nurse knows that an antitussive cough medication would be the best choice for which patient? a A patient with a productive cough b A patient with chronic paranasal sinusitis c A patient who has had recent abdominal surgery d A patient who has influenza

4. c

4 A patient is receiving dobutamine for shock and is complaining of feeling more "skipping beats" than yesterday. What will the nurse do next? a Monitor for other signs of a therapeutic response to the drug. b Titrate the drug to a higher dose to reduce the palpitations. c Discontinue the dobutamine immediately. d Assess the patient's vital signs and cardiac rhythm.

4. d

4 A patient who has been newly diagnosed with HIV has many questions about the effectiveness of drug therapy. After a teaching session, which statement by the patient reflects a need for more education? a "I will be monitored for side effects and improvements while I'm taking this medicine." b "These drugs do not eliminate the HIV, but hopefully the amount of virus in my body will be reduced." c "There is no cure for HIV." d "These drugs will eventually eliminate the virus from my body."

4. d

4 A patient who has recently had a myocardial infarction (MI) has started therapy with a beta blocker. The nurse explains that the main purpose of the beta blocker for this patient is to a cause vasodilation of the coronary arteries. b prevent hypertension. c increase conduction through the SA node. d protect the heart from circulating catecholamines.

4. d

4 The nurse is preparing an infusion for a patient who has a deficiency in clotting factors. Which type of infusion is most appropriate? a Albumin 5% b Packed RBCs c Whole blood d Fresh frozen plasma

4. d

5 A patient who is taking oral tetracycline complains of heartburn and requests an antacid. Which action by the nurse is correct? a Give the tetracycline, but delay the antacid for 1 to 2 hours. b Give the antacid, but delay the tetracycline for at least 4 hours. c Administer both medications together. d Explain that the antacid cannot be given while the patient is taking the tetracycline.

5. a

5 During drug therapy with basiliximab, the nurse monitors for signs of cytokine release syndrome, which results in a fever, dyspnea, and general malaise. b neurotoxicity and peripheral neuropathy. c hepatotoxicity with jaundice. d thrombocytopenia with increased bleeding tendencies.

5. a

5 The nurse is giving medications to a patient. Which drug or drug class, when administered with lithium, increases the risk for lithium toxicity? a Thiazides b levofloxacin c calcium citrate d Beta blockers

5. a

5 The nurse is reviewing the history of a patient who will be starting the triptan sumatriptan (Imitrex) as part of treatment for migraine headaches. Which condition, if present, may be a contraindication to triptan therapy? a Cardiovascular disease b Chronic bronchitis c History of renal calculi d Diabetes mellitus type 2

5. a

5 After the patient takes a dose of an inhaled corticosteroid, such as fluticasone (Flovent), what is the most important action the patient needs to do next? a Hold the breath for 60 seconds. b Rinse out the mouth with water. c Follow the corticosteroid with a bronchodilator inhaler, if ordered. d Repeat the dose in 15 minutes if the patient feels short of breath.

5. b

5 When a drug is characterized as having a negative chronotropic effect, the nurse knows to expect which effect? a Reduced blood pressure b Decreased heart rate c Decreased ectopic beats d Increased force of cardiac contractions

5. b

5 A 28-year-old is in the urgent care center after stepping on a rusty tent nail. The nurse evaluates the patient's immunity status and notes that the patient thinks she had her last tetanus booster about 10 years ago, just before starting college. Which immunization would be most appropriate at this time? a Immunoglobulin intravenous (Gammar-P IV) b DTaP (Daptacel) (diphtheria, tetanus, and acellular pertussis) c Tdap (Adacel) (diphtheria, tetanus, and acellular pertussis) d No immunizations are necessary at this time.

5. c

5 After surgery for organ transplantation, a patient is receiving ganciclovir, even though he does not have a viral infection. Which statement best explains the rationale for this medication therapy? a Ganciclovir is used to prevent potential exposure to the HIV virus. b This medication is given prophylactically to prevent influenza A infection. c Ganciclovir is given to prevent CMV infection. d The drug works synergistically with antibiotics to prevent superinfections.

5. c

5 During the assessment of a patient about to receive a cholinergic-blocking drug, the nurse will determine whether the patient is taking any drugs that may potentially interact with the anticholinergic, including: a opioids, such as morphine sulfate. b antibiotics, such as penicillin. c tricyclic antidepressants, such as amitriptyline. d anticonvulsants, such as phenobarbital.

5. c

5 The nurse is discussing therapy with clomiphene (Clomid) with a husband and wife who are considering trying this drug as part of treatment for infertility. It is important that they be informed of which possible effect of this drug? a Increased menstrual flow b Increased menstrual cramping c Multiple pregnancy (twins or more) d Sedation

5. c

5 The nurse is reviewing the orders for a newly admitted patient and sees an order for edrophonium (Tensilon). The nurse expects that this drug is ordered for which reason? a To reduce symptoms and delay the onset of Alzheimer's disease b To treat the symptoms of myasthenia gravis c To aid in the diagnosis of myasthenia gravis d To reverse the effects of nondepolarizing neuromuscular blocking drugs after surgery

5. c

5 Which teaching is appropriate for a patient who is taking an inhaled glucocorticoid for asthma? a "Exhale while pushing in on the canister of the inhaler." b "Blow your nose after taking the medication." c "Rinse your mouth thoroughly after taking the medication." d "Do not eat immediately after taking the medication."

5. c

5 A hospitalized patient is complaining of having difficulty sleeping. Which action will the nurse take first to address this problem? a Administer a sedative-hypnotic drug if ordered. b Offer tea made with the herbal preparation valerian. c Encourage the patient to exercise by walking up and down the halls a few times if tolerated. d Provide an environment that is restful, and reduce loud noises.

5. d

5 A patient is taking a decongestant to help reduce symptoms of a cold. The nurse will instruct the patient to observe for which possible symptom, which may indicate an adverse effect of this drug? a Increased cough b Dry mouth c Slower heart rate d Heart palpitations

5. d

6 When the nurse is administering a proton pump inhibitor (PPI), which actions by the nurse are correct? (Select all that apply.) a Giving the PPI on an empty stomach b Giving the PPI with meals c Making sure the patient does not crush or chew the capsules d Instructing the patient to open the capsule and chew the contents for best absorption e Administering the PPI only when the patient complains of heartburn

6. a, c

6 A patient is taking an alpha blocker as treatment for benign prostatic hyperplasia. The nurse will monitor for which potential drug effects? (Select all that apply.) a Orthostatic hypotension b Increased blood pressure c Increased urine flow d Headaches e Bradycardia

6. a, c, d

6 The nurse is monitoring a patient who is receiving an infusion of a beta-adrenergic agonist. Which adverse effects may occur with this infusion? (Select all that apply.) a Mild tremors b Bradycardia c Tachycardia d Palpitations e Drowsiness f Nervousness

6. a, c, d, f

6 The nurse is teaching a patient about the inhaler Advair (salmeterol/fluticasone). Which statements by the patient indicate a correct understanding of this medication? Select all that apply. a "I will rinse my mouth with water after each dose." b "I need to use this inhaler whenever I feel short of breath, but not less than 4 hours between doses." c "This medication is taken twice a day, every 12 hours." d "I can take this inhaler if I get short of breath while exercising." e "I will call my doctor if I notice white patches inside my mouth."

6. a, c, e

6 When assessing a patient who is to begin therapy with an immunosuppressant drug, the nurse recognizes that such drugs should be used cautiously in patients with which condition(s)? (Select all that apply.) a Pregnancy b Glaucoma c Anemia d Myalgia e Renal dysfunction f Hepatic dysfunction

6. a, d, f

6 A patient calls the clinic because she realized she missed one dose of an oral contraceptive. Which statement from the nurse is appropriate? (Select all that apply.) a "Go ahead and take the missed dose now, along with today's dose." b "Don't worry, you are still protected from pregnancy." c "Please come to the clinic for a reevaluation of your therapy." d "Wait 7 days, and then start a new pack of pills." e "You will need to use a backup form of contraception concurrently for 7 days."

6. a, e

6 The nurse is administering an IV solution that contains potassium chloride to a patient in the critical care unit who has a severely decreased serum potassium level. Which action(s) by the nurse are appropriate? (Select all that apply.) a Administer the potassium by slow IV bolus. b Administer the potassium at a rate no faster than 20 mEq/hr. c Monitor the patient's cardiac rhythm with a heart monitor. d Use an infusion pump for the administration of IV potassium chloride. e Administer the potassium IV push.

6. b, c, d

6 The nurse is giving an antihistamine and will observe the patient for which side effects? (Select all that apply.) a Hypertension b Dizziness c "Hangover" effect d Drowsiness e Tachycardia f Dry mouth

6. b, c, d, f

6 The nurse is teaching a patient about treatment with an SSRI antidepressant. Which teaching considerations are appropriate? (Select all that apply.) a The patient should be told which foods contain tyramine and instructed to avoid these foods. b The patient should be instructed to use caution when standing up from a sitting position. c The patient should not take any products that contain the herbal product St. John's wort. d This medication should not be stopped abruptly. e Drug levels may become toxic if dehydration occurs. f The patient should be told to check with the prescriber before taking any over-the-counter medications.

6. b, c, d, f

6 During long-term corticosteroid therapy, the nurse will monitor the patient's laboratory results for adverse effects, such as: (Select all that apply.) a Increased serum potassium levels b Decreased serum potassium levels c Increased sodium levels d Decreased sodium levels e Hyperglycemia f Hypoglycemia

6. b, c, e

6 The nurse is reviewing the use of multidrug therapy for HIV with a patient. Which statements are correct regarding the reason for using multiple drugs to treat HIV? (Select all that apply.) a The combination of drugs has fewer associated toxicities. b The use of multiple drugs is more effective against resistant strains of HIV. c Effective treatment results in reduced T-cell counts. d The goal of this treatment is to reduce the viral load. e This type of therapy reduces the incidence of opportunistic infections.

6. b, d, e

6 When giving intravenous cholinergic drugs, the nurse must watch for symptoms of a cholinergic crisis, such as: (Select all that apply.) a peripheral tingling. b hypotension. c dry mouth. d syncope. e dyspnea. f tinnitus.

6. b, d, e

6 A patient has been given a prescription for transdermal scopolamine patches (Transderm-Scōp) for motion sickness for use during a vacation cruise. The nurse will include which instructions? (Select all that apply.) a "Apply the patch as soon as you board the ship." b "Apply the patch 3 to 4 hours before boarding the ship." c "The patch needs to be placed on a nonhairy area on your upper chest or upper arm." d "The patch needs to be placed on a nonhairy area just behind your ear." e "Change the patch every 3 days." f "Rotate the application sites."

6. b, d, e, f

6 The nurse is reviewing medication therapy with the parents of an adolescent with ADHD. Which statement is correct? (Select all that apply.) a "Be sure to have your child blow his nose before administering the nasal spray." b "This medication is used only when symptoms of ADHD are severe." c "The last dose should be taken 4 to 6 hours before bedtime to avoid interference with sleep." d "Be sure to contact the physician right away if you notice expression of suicidal thoughts." e "We will need to check your child's height and weight periodically to monitor physical growth." f "If adverse effects become severe, stop the medication for 3 to 4 days."

6. c, d, e

6 Which considerations are important for the nurse to remember when administering a benzodiazepine as a sedative-hypnotic drug? (Select all that apply.) a These drugs are intended for long-term management of insomnia. b The drugs can be administered safely with other CNS depressants for insomnia. c The dose needs to be given about 1 hour before the patient's bedtime. d The drug is used as a first choice for treatment of sleeplessness. e The patient needs to be evaluated for the drowsiness that may occur the morning after a benzodiazepine is taken.

6. c, e

6 The nurse is providing teaching after an adult receives a booster immunization. Which adverse reactions should be reported immediately to the health care provider? (Select all that apply.) a Swelling and redness at the injection site b Fever of 100° F (37.8° C) c Joint pain d Heat over the injection site e Rash over the arms, back, and chest f Shortness of breath

6. c, e, f

Patient teaching for a patient being discharged on a beta blocker includes which statement? A. "If you take your pulse and it is less than 60, hold your medicine and call your health care provider for instructions." B. "If you become dizzy, do not take your medication for 2 days and then restart on the third day." C. "This medication may make you fatigued; increasing caffeine in your diet may help alleviate this problem." D. "Increase intake of green leafy vegetables to prevent bleeding problems that can be caused by this medication."

A. "If you take your pulse and it is less than 60, hold your medicine and call your health care provider for instructions." Rationale Beta blockers have a negative chronotropic effect and could cause symptomatic bradycardia and/or heart block. The physician should be consulted before administering to a patient with bradycardia (heart rate <60 beats/min).

When educating patients on immunosuprressants, what information would the nurse include in the teaching? (Select all that apply.) A. "Never stop taking these medications without being instructed by the prescribing provider." B. "Over-the-counter medication are OK to take as needed." C. "You must take all medications exactly as prescribed." D. "Medications must be taken at the correct time every time to avoid interactions."

A. "Never stop taking these medications without being instructed by the prescribing provider." C. "You must take all medications exactly as prescribed." D. "Medications must be taken at the correct time every time to avoid interactions." Rationale Immunosuppressants must be taken exactly as directed and at the exact times and with the exact foods. Adherence to dosing schedules can be very difficult for patients because they are taking multiple medications that must be taken at different times throughout the day. Patients should never stop taking their immunosuppressants without being told to do so by their transplant doctor.

A patient taking oral contraceptives is being treated for a urinary tract infection with antibiotics. Which information should the nurse include as education related to the oral contraceptives? A. "Use an alternative method of birth control this month during antibiotic use." B. "The urinary tract infection can be passed to your partner, so he should be screened." C. "Be sure to complete the full course of antibiotics." D. "There is no interaction of oral contraceptives and antibiotics."

A. "Use an alternative method of birth control this month during antibiotic use." Rationale When a patient takes oral contraceptives and is prescribed an antibiotic, the oral contraceptive effectiveness can be decreased and an alternative method of birth control should be used for the month.

A patient is making an appointment for allergy testing. The nurse instructs the patient not take what class of medications at least 4 days before allergy testing? A. Antihistamines B. Decongestants C. Bronchodilators D. Antitussives

A. Antihistamines Rationale An allergist will usually recommend discontinuation of antihistamine drug therapy at least 4 days before allergy testing.

The nurse is discussing management of symptoms of an upper respiratory tract infection. Which classes of medications are often used in treating the symptoms of upper respiratory tract infections? (Select all that apply.) A. Antihistamines B. Nasal decongestants C. Antitussives D. Expectorants E. Antibiotics

A. Antihistamines B. Nasal decongestants C. Antitussives D. Expectorants Rationale Treatment of the common symptoms of upper respiratory tract infections involves the combined use of antihistamines, nasal decongestants, antitussives, and expectorants. Upper respiratory tract infections are often viral or allergic in nature and the symptoms would not be managed with antibiotics.

Selective serotonin reuptake inhibitors (SSRIs) and tricyclic antidepressants (TCAs) both function by which mechanism? A. Blocking the reuptake of neurotransmitters at nerve endings B. Increasing alertness levels in the brain C. Decreasing levels of epinephrine and serotonin at nerve endings D. Increasing the placebo effect

A. Blocking the reuptake of neurotransmitters at nerve endings Rationale SSRIs block the reuptake of serotonin. TCAs block the reuptake of norepinephrine and serotonin.

Before administering an immunosuppressant drug, the nurse should perform which assessments? (Select all that apply.) A. Blood urea nitrogen and creatinine levels B. Level of consciousness C. Blood pressure and pulse D. Hepatic enzymes

A. Blood urea nitrogen and creatinine levels B. Level of consciousness C. Blood pressure and pulse D. Hepatic enzymes Rationale Adverse reactions to immunosuppressants include neurotoxicity, renal toxicity, hepatotoxicity, and hypertension.

The nurse is providing care to a patient prescribed a nonselective adrenergic agonist bronchodilator. Which medical diagnosis on this patient's chart would alert the nurse to question this order? A. Coronary artery disease B. Chronic obstructive pulmonary disease (COPD) C. Hyperkalemia D. Premature labor

A. Coronary artery disease Rationale Nonselective adrenergic agonist bronchodilators stimulate beta1 receptors in the heart and beta2 receptors in the lungs. Stimulation of beta1 receptors can increase heart rate and contractility, increasing oxygen demand. This increased oxygen demand may lead to angina or myocardial ischemia in patients with coronary artery disease.

Which conditions is aminoglutethimide used to treat? (Select all that apply.) A. Cushing's syndrome B. Testicular cancer C. Adrenal cancer D. Metastatic breast cancer E. Thyroid cancer

A. Cushing's syndrome C. Adrenal cancer D. Metastatic breast cancer Rationale Aminoglutethimide is an adrenal steroid inhibitor. Aminoglutethimide obstructs the normal actions of the adrenal cortex by inhibiting the conversion of cholesterol into adrenal corticosteroids. Aminoglutethimide is used in the treatment of Cushing's syndrome, metastatic breast cancer, and adrenal cancer.

The nurse monitors a patient prescribed dicyclomine (Bentyl) for which therapeutic effect? A. Decrease in gastrointestinal motility B. Decrease in urinary frequency C. Increase in heart rate D. Increase in blood pressure

A. Decrease in gastrointestinal motility Rationale Dicyclomine is an antispasmodic cholinergic blocker used to decrease gastrointestinal motility in patients with functional gastrointestinal disorders such as irritable bowel syndrome.

The nurse would question the order for estrogen replacement therapy in a patient with a history of which condition? A. Deep vein thrombosis B. Vaginal bleeding C. Weight loss D. Dysmenorrhea

A. Deep vein thrombosis Rationale Increased coagulation and risk of deep vein thrombosis are side effects of hormone replacement therapy.

The nurse is discussing use of antitussive medications with a patient. What common side effect does the nurse include in the patient teaching? A. Drowsiness and dizziness B. Diarrhea and abdominal cramping C. Tremors and anxiety D. Bradycardia and increased lacrimation

A. Drowsiness and dizziness Rationale Antitussive medications also affect the central nervous system, thus causing drowsiness and dizziness.

The nurse monitors a patient taking an antipsychotic medication for extrapyramidal side effects. What is the nurse assessing for in the patient? A. Dystonia B. Orthostatic hypotension C. Dry mouth and constipation D. Neuroleptic malignant syndrome

A. Dystonia Rationale Dystonia, an impairment of muscle tone, is the only extrapyramidal side effect listed. The other side effects also occur but are not extrapyramidal effects.

5. The nurse is caring for a patient with an acute renal insufficiency and thrombocytopenia. Along with platelet transfusions, the nurse would expect to administer which substance to increase deficient clotting factors in this patient? A. Fresh frozen plasma B. Albumin C. Plasma protein factors D. Whole blood

A. Fresh frozen plasma Fresh frozen plasma is indicated to increase clotting factors in patients with a known deficiency. Albumin and plasma protein factors do not contain clotting factors. Although whole blood does contain the same ingredients as fresh frozen plasma, the amount of volume that must be administered to give the patient the necessary clotting factors may be prohibitive in a patient with renal insufficiency.

Before administration of any antiviral medication, what nursing care would be performed? (Select all that apply.) A. Head-to-toe physical assessment B. Documentation of known allergies C. History of medication use D. Monitoring for adverse effects E. Baseline vital signs

A. Head-to-toe physical assessment B. Documentation of known allergies C. History of medication use E. Baseline vital signs Rationale Before administering an antiviral drug, perform a thorough head-to-toe physical assessment and take a medical and medication history. Document any known allergies before use of these and any other medications. Also assess the patient's baseline vital signs because of the profound effects of viral illnesses on physiologic status, especially if the patient is immunocompromised. Assess and document any contraindications, cautions, and drug interactions associated with all of the antiviral drugs. Monitoring for adverse effects would occur after the medication has been administered.

Your patient is receiving dobutamine as a continuous infusion. Titration of this medication is based upon which factors? (Select all that apply.) A. Heart rate B. Blood pressure C. Urine output D. Liver enzymes E. Respiratory rate

A. Heart rate B. Blood pressure C. Urine output Rationale The major therapeutic effect of dobutamine is to increase cardiac output. Cardiac output is reflected in the patient's heart rate, blood pressure, and urine output.

Beta blockers are used to treat which disorders? (Select all that apply.) A. Hypertension B. COPD C. Angina pectoris D. Cardiac dysrhythmias E. Raynaud's disease

A. Hypertension C. Angina pectoris D. Cardiac dysrhythmias Rationale Beta blockers are effective in treating hypertension (secondary to negative inotropic effects), angina pectoris (decreases cardiac workload when decreasing heart rate and contractility), and cardiac dysrhythmias (decreasing heart rate and conductivity). Beta blockers can exacerbate COPD as they block beta2 receptors and subsequent bronchodilation. Raynaud's disease may be treated with alpha, not beta, blockers.

The nurse is aware that viruses can enter the body through various routes. By which methods can viruses enter the body? (Select all that apply.) A. Inhalation through the respiratory tract B. Ingestion via the gastrointestinal (GI) tract C. Transplacentally from mother to infant D. Through an animal bite

A. Inhalation through the respiratory tract B. Ingestion via the gastrointestinal (GI) tract C. Transplacentally from mother to infant D. Through an animal bite Rationale Viruses can enter the body through at least four routes: inhalation through the respiratory tract, ingestion via the GI tract, transplacentally via mother to infant, and inoculation via skin or mucous membranes. The inoculation route can take several forms, including sexual contact, blood transfusions, sharing of syringes or needles, organ transplantation, and animal bites (including human, animal, insect, spider, and others).

When assessing for side effects expected in a patient taking analeptics, the nurse would monitor for which effect? A. Insomnia B. Bradycardia C. Hypotension D. Decreased mental alertness

A. Insomnia Rationale Analeptics are CNS stimulants, which tend to "speed up" body systems. Adverse effects include hypertension, tachycardia, angina, anxiety, insomnia, headache, tremor, blurred vision, increased metabolic rate, gastrointestinal distress, and dry mouth.

Before administering an antileukotriene medication, the nurse would assess the patient for allergies to which substance? (Select all that apply.) A. Lactose B. Povidone C. Cellulose D. Chlorhexedine

A. Lactose B. Povidone C. Cellulose Rationale Allergy to povidone, lactose, titanium dioxide, or cellulose derivatives is important to note because these are inactive ingredients in antileukotriene drugs.

Which laboratory value would the nurse assess before administering zafirlukast (Accolate) to a patient? A. Liver enzymes B. Cardiac enzymes C. Renal function tests D. Complete blood count

A. Liver enzymes Rationale Since zafirlukast may lead to liver dysfunction, liver enzyme levels should be monitored regularly, especially early in the course of therapy.

4. What condition will the nurse monitor for in a patient using sodium bicarbonate to treat gastric hyperacidity? A. Metabolic alkalosis B. Metabolic acidosis C. Hyperkalemia D. Hypercalcemia

A. Metabolic alkalosis Rationale Solutions containing sodium bicarbonate (a base) can cause metabolic alkalosis. Serum potassium and serum calcium would decrease with alkalosis, not increase.

The anthrax vaccine is recommended for which groups of people? (Select all that apply.) A. Military personnel B. Veterinarians C. Workers who process imported animal hair D. Emergency department health care providers

A. Military personnel B. Veterinarians C. Workers who process imported animal hair Rationale People at risk for exposure to the anthrax bacterium include military personnel, veterinarians, and workers who process imported animal hair.

Which types of antiviral drugs are used to treat HIV infection? (Select all that apply.) A. Nonnucleoside reverse transcriptase inhibitors B. Protease inhibitors C. Reverse transcriptase inhibitors D. Fusion inhibitors E. Neuraminidase inhibitors

A. Nonnucleoside reverse transcriptase inhibitors B. Protease inhibitors C. Reverse transcriptase inhibitors D. Fusion inhibitors Rationale Neuraminidase inhibitors are used in the treatment of the influenza virus.

To prevent oral candidiasis, it is most important for the nurse to teach a patient using a steroid inhaler to perform which action? A. Rinse the mouth after each use. B. Minimize use of an inhaler to every other day. C. Swish and swallow with mycostatin after each use. D. Report any gingival irritation to the health care provider.

A. Rinse the mouth after each use. Rationale It is most important to teach patients to rinse their mouth after each use of a steroid inhaler to prevent the occurrence of oral candidiasis, a fungal infection. Mycostatin is not routinely used to prevent this infection unless the patient is immunocompromised. Reporting irritation once it has occurred does not prevent the infection. Minimizing the use of the inhaler to every other day negates its therapeutic effect.

The priority nursing diagnosis for a patient taking metoprolol (Lopressor) would be A. Risk for decreased cardiac tissue perfusion related to effects of medication. B. Acute confusion related to adverse central nervous system effects of the drug. C. Deficient knowledge related to therapeutic regimen. D. Risk for injury related to possible side effects of the adrenergic blockers.

A. Risk for decreased cardiac tissue perfusion related to effects of medication. Rationale Using the ABCs of prioritization, Risk for decreased cardiac tissue perfusion puts the patient at highest risk. Although the other nursing diagnoses are pertinent, they are not the priority.

Discharge teaching to a patient receiving a beta-agonist bronchodilator should emphasize reporting which side effect? A. Tachycardia B. Nonproductive cough C. Hypoglycemia D. Sedation

A. Tachycardia Rationale A beta-agonist bronchodilator stimulates the beta receptors of the sympathetic nervous system, resulting in tachycardia, bronchodilation, hyperglycemia, and increased alertness.

What is the action of histamine2-receptor antagonists? A. They compete with histamine for binding sites on the parietal cells. B. They irreversibly bind to H+/K+ ATPase. C. They cause a decrease in stomach pH. D. They decrease signs and symptoms of allergies related to histamine release.

A. They compete with histamine for binding sites on the parietal cells. Rationale Histamine receptor-blocking drugs decrease gastric acid by competing with histamine for binding sites on the parietal cells.

When discussing glucocorticoids, what statement is accurate in relation to the action of these medications? A. They influence carbohydrate, fat, and protein metabolism. B. They are produced in decreased amounts during times of stress. C. They decrease serum sodium and glucose levels. D. They stimulate defense mechanisms to produce immunity.

A. They influence carbohydrate, fat, and protein metabolism. Rationale Glucocorticoids play a major role in carbohydrate, lipid, and protein metabolism within the body. They are produced in increasing amounts during stress, increase sodium and glucose levels, and suppress the immune system.

The nurse is providing education to a patient on the primary uses of cholinergic drugs. Which uses would the nurse include in the teaching? (Select all that apply.) A. To stimulate peristalsis B. To elevate heart rate C. To decrease intraocular pressure D. To stimulate bladder emptying E. To dilate pulmonary airways

A. To stimulate peristalsis C. To decrease intraocular pressure D. To stimulate bladder emptying Rationale Cholinergic drugs are used primarily for their effects on the GI tract, bladder, and eye. These drugs stimulate the intestine and bladder, which results in increased gastric secretions, GI motility, and urinary frequency. They also stimulate constriction of the pupil, or miosis. This helps decrease intraocular pressure. In addition, cholinergic drugs cause increased salivation and sweating. Cardiovascular effects include reduced heart rate and vasodilation. Pulmonary effects include causing the bronchi of the lungs to constrict and the airways to narrow.

The nurse would question an order for steroids in a patient with which condition? A. Uncontrolled diabetes mellitus B. Rheumatoid arthritis C. Septic shock D. Exacerbation of chronic obstructive pulmonary disease (COPD)

A. Uncontrolled diabetes mellitus Rationale A common side effect of steroid therapy is hyperglycemia; therefore uncontrolled diabetes mellitus is a contraindication to steroid therapy.

Which statement most accurately describes the pharmacodynamics of vaccines? A. Vaccines work by stimulating the humoral immune system. B. Vaccines provide passive immunity. C. Vaccines work by suppressing immunoglobulins. D. Vaccines prevent the formation of antibodies against a specific antigen.

A. Vaccines work by stimulating the humoral immune system. Rationale Vaccines work by stimulating the humoral immune system, which synthesizes immunoglobulins. They also stimulate the formation of antibodies against their specific antigen, providing active immunity.

The nurse is educating a patient who has been prescribed methylphenidate for narcolepsy about the drug's adverse effects. What potential adverse effects would the nurse include in the patient teaching? (Select all that apply.) A. Weight Loss B. Headache C. Insomnia D. Decreased blood pressure E. Increased appetite

A. Weight Loss B. Headache C. Insomnia Rationale Adverse effects of methylphenidate on the cardiovascular system include increased heart rate and blood pressure. Other adverse effects include angina, anxiety, insomnia, headache, tremor, blurred vision, increased metabolic rate, gastrointestinal (GI) distress, dry mouth, and worsening of or new onset of psychiatric disorders, including mania, psychoses, or aggression.

When assessing for cardiovascular effects of an adrenergic (sympathomimetic) drug, the nurse understands that these drugs produce A. a positive inotropic, positive chronotropic, and positive dromotropic effect. B. a positive inotropic, negative chronotropic, and negative dromotropic effect. C. a negative inotropic, positive chronotropic, and positive dromotropic effect. D. a negative inotropic, negative chronotropic, and negative dromotropic effect.

A. a positive inotropic, positive chronotropic, and positive dromotropic effect. Rationale Adrenergic stimulation of the beta1-adrenergic receptors on the myocardium and in the conduction system of the heart results in an increased heart rate (positive chronotropic effect), increased contractility (positive inotropic effect), and increased conductivity (positive dromotropic effect).

The mother of a child with attention deficit hyperactivity disorder (ADHD) who has been prescribed methylphenidate (Ritalin) expresses concern regarding the use of a controlled substance to treat her child and asks if there are any other options. The nurse's response is based on the knowledge that an option for treatment for ADHD might include which non-controlled central nervous system (CNS) stimulant? A. atomoxetine (Strattera) B. dextroamphetamine sulfate (Dexedrine) C. methylphenidate (Concerta) D. amphetamine aspartate (Adderall)

A. atomoxetine (Strattera) Rationale Atomoxetine (Strattera) is not a controlled substance as it lacks addictive properties, unlike amphetamines and phenidates.

. The nurse is admitting a patient with a history of angina and hypertension who is currently experiencing moderate heart failure. The patient's current medication regimen includes digoxin (Lanoxin), furosemide (Lasix), and quinapril (Accupril). Which medication would be most beneficial for the health care provider to add to this patient's treatment plan? A. carvedilol (Coreg) B. propranolol (Inderal) C. esmolol (Brevibloc) D. sotalol (Betapace)

A. carvedilol (Coreg) Rationale Carvedilol (Coreg), a combined alpha1 and beta blocker, has been shown to slow the progression of heart failure and decrease the frequency of hospitalization in patients with mild to moderate (Class II or III) heart failure. Carvedilol is most commonly added to digoxin, furosemide, and angiotensin-converting enzyme (ACE) inhibitors when used to treat heart failure.

Side effects to expect from anticholinergic (parasympatholytic) drugs, such as atropine, include (Select all that apply.) A. dilated pupils. B. urinary retention. C. dry mouth. D. diarrhea. E. increased sweating.

A. dilated pupils. B. urinary retention. C. dry mouth. Rationale Anticholinergic drugs block the effects of the parasympathetic nervous system, producing sympathetic nervous system effects. These include mydriasis (dilated pupils), decreased bladder contraction, and decreased oral secretions. The effect on the gastrointestinal system would be to decrease gastrointestinal motility, not cause diarrhea. Sweating would decrease as a result of anticholinergic drugs.

The physician has ordered dopamine to treat the patient's hypovolemic shock secondary to severe blood loss. For the medication to be effective, the physician must also order A. fluid replacement. B. beta-stimulating drugs. C. antibodies. D. fluid restriction.

A. fluid replacement. Rationale Dopamine increases blood pressure secondary to vasoconstriction, which has a limited effect if there is not enough volume within the circulatory system.

A patient asks the nurse about a new drug advertised on television. The patient wants to know if Ambien would be better for her to use than her current medication, Restoril, for periodic insomnia. The nurse's response is based on knowledge that zolpidem (Ambien) (Select all that apply.) A. is less likely to cause grogginess in the morning. B. is a pregnancy category C medication. C. is contraindicated with asthma. D. should be limited to 7 to 10 days of treatment.

A. is less likely to cause grogginess in the morning. B. is a pregnancy category C medication. D. should be limited to 7 to 10 days of treatment. Rationale Zolpidem (Ambien) is a short-acting nonbenzodiazepine hypnotic drug. It is indicated for the short-term treatment of insomnia and should be limited to 7 to 10 days of treatment; it is pregnancy category C. Zolpidem is less likely to produce grogginess in the morning after the medication has been taken when compared with other hypnotic benzodiazepines. It is not currently contraindicated in cases of asthma.

Before administering intravenous basiliximab (Simulect), the nurse would anticipate giving which medication? A. methylprednisolone sodium (Solu-Medrol) B. diphenhydramine (Benadryl) C. acetaminophen (Tylenol) D. meperidine (Demerol)

A. methylprednisolone sodium (Solu-Medrol) Rationale Methylprednisolone sodium is administered 30 minutes before basiliximab injection to prevent or minimize acute allergic-type reactions associated with this medication.

1. The nurse will teach patients that antacids are effective in treatment of hyperacidity because they A. neutralize gastric acid. B. decrease stomach motility. C. decrease gastric pH. D. decrease duodenal pH.

A. neutralize gastric acid. Rationale Antacids work by neutralizing gastric acid, which would cause an increase in pH. They do not affect gastric motility.

When providing health promotion teaching at a senior citizen center, the nurse would include information about which medication used to decrease the duration of influenza A and B? A. oseltamivir (Tamiflu) B. ganciclovir (Cytovene) C. enfuvirtide (Fuzeon) D. indinavir (Crixivan)

A. oseltamivir (Tamiflu) Rationale Oseltamivir (Tamiflu) and zanamivir (Relenza) are active against influenza virus types A and B and have been shown to reduce the duration of influenza infection by several days.

Ergot alkaloids such as ergotamine tartrate (Ergostat) exert a therapeutic effect by causing A. vasoconstriction. B. vasodilation. C. blockade of the beta2 receptors. D. simulation of the alpha receptors.

A. vasoconstriction. Rationale Ergot alkaloids block alpha2 receptors, causing vasoconstriction. They are useful in treating vascular headaches caused by vasodilation of vessels in the brain.

A patient is being switched from amitriptyline (Elavil) to citalopram (Celexa). Which statement made by the patient reflects understanding of patient education? A. "I can just stop taking my Elavil and start taking the Celexa as ordered." B. "I will not get as dizzy when I change positions after I switch medications." C. "The doctor is switching me to this medication because it is less expensive but just as effective." D. "I will need to limit my intake of cheese when taking Celexa to prevent a rise in my blood pressure."

B. "I will not get as dizzy when I change positions after I switch medications." Rationale Citalopram, an SSRI, produces minimal anticholinergic and cardiovascular side effects.

Which statement by a patient best demonstrates an understanding of the teaching on flunisolide (AeroBid)? A. "I will take two puffs to treat an acute asthma attack." B. "I will rinse my mouth with water after each use." C. "I will immediately stop taking my oral prednisone as soon as I start using the AeroBid." D. "I will not use my albuterol inhaler while I am taking AeroBid."

B. "I will rinse my mouth with water after each use." Rationale Flunisolide is an inhaled corticosteroid. Rinsing the mouth will help prevent oral candidal infections. It is not used to treat an acute asthma attack and should be taken with the patient's bronchodilator medications. If the patient is taking oral prednisone, it needs to be tapered off to prevent acute adrenal crisis since flunisolide is minimally absorbed systemically.

7. Which statement demonstrates understanding of patient teaching regarding the use of histamine2-receptor antagonists? A. "Since I am taking this medication, it is OK for me to eat spicy foods." B. "Smoking decreases the effects of this medication, so I should look into cessation programs." C. "I should take this medication 1 hour after each meal in order to maximally decrease gastric acidity." D. "I should decrease bulk and fluids in my diet to prevent diarrhea."

B. "Smoking decreases the effects of this medication, so I should look into cessation programs." Rationale Patients taking histamine2-receptor-blocking drugs should avoid spicy foods, extremes in temperatures, alcohol, and smoking. They should also increase bulk and fluids in the diet to prevent constipation. Cimetidine should be taken with meals, while famotidine can be taken without regard to meals.

What teaching would the nurse provide to a patient receiving tetanus toxoid? A. "Increase fluid and fiber in your diet to prevent constipation." B. "Soreness at the injection site is a common reaction." C. "You will have lifetime immunity from this injection." D. "This medication must be repeated weekly for four weeks."

B. "Soreness at the injection site is a common reaction." Rationale Myalgia at the injection site is a common side effect of tetanus toxoid.

Which statement by the patient demonstrates understanding of action or use of beclomethasone diproprionate (Beconase)? A. "I will need to taper off the medication to prevent acute adrenal crisis." B. "This medication will help prevent the inflammatory response of my allergies." C. "I will need to monitor my blood sugar more closely because it may increase secondary to Beconase." D. "I only need to take this medication when my symptoms get bad."

B. "This medication will help prevent the inflammatory response of my allergies." Rationale Beclomethasone diproprionate is a steroid spray administered nasally. It is used to prevent allergy symptoms. Its effect is localized, and therefore the patient does not have systemic side effects with normal use and does not have to worry about weaning down the medication as with oral corticosteroids.

An adult patient presents with symptomatic bradycardia. The nurse prepares to administer which dose of atropine intravenously? A. 0.3 mg B. 0.5 mg C. 1.25 mg D. 2 mg

B. 0.5 mg Rationale The recommended dose of atropine to treat symptomatic bradycardia is 0.5 to 1 mg.

The current immunization for tetanus and diphtheria toxoids and pertussis, Tdap, is administered to people in which age range? A. Under 6 years of age B. 11 years of age and older C. Any age range D. In the first two years of life

B. 11 years of age and older Rationale Currently DTaP is the preferred preparation for primary and booster immunization against these diseases in children from 6 weeks to 6 years of age, unless use of the pertussis component is contraindicated. Tdap is the recommended vaccine for adolescents and adults, those over the age of 11.

. The nurse has an order for a patient to receive prednisone for contact dermatitis. What condition in the patient would alert the nurse to question the order? A. Asthma B. AIDS C. COPD D. Multiple sclerosis

B. AIDS Rationale Because of their immunosuppressant properties, glucocorticoids are often avoided in the presence of any serious infection, including septicemia, systemic fungal infections, and varicella. These drugs would be contraindicated in patients with active AIDS infection as it could potentially decrease the immune system further. One exception is tuberculous meningitis, for which glucocorticoids may be used to prevent inflammatory central nervous system damage.

Midazolam (Versed) has been ordered for a patient to be administered by injection 30 minutes prior to a colonoscopy. The nurse informs the patient that one of the most common side effects of this medication is which effect? A. Decreased heart rate B. Amnesia C. Constipation D. Dry mouth

B. Amnesia Rationale Versed is known to cause amnesia and anxiolysis as well as sedation and is therefore commonly used prior to certain procedures.

A nurse would monitor older adults who are prescribed a benzodiazepine for treatment of insomnia for which potential side effect? A. Hallucinations B. Ataxia C. Alertness D. Dyspnea

B. Ataxia Rationale Benzodiazepine doses for children and the elderly should be small with gradual increases to avoid ataxia and excessive sedation. Thus, these patients should be closely monitored for these adverse effects.

Which statement correctly identifies the pharmacodynamics of CNS stimulants? A. CNS stimulants decrease the production of excitatory neurotransmitters. B. CNS stimulants increase release of and block reuptake of neurotransmitters. C. CNS stimulants block the activity of inhibitory neurons. D. CNS stimulants enhance the effects of phosphodiesterase and subsequent breakdown of cyclic adenosine monophosphate (cAMP).

B. CNS stimulants increase release of and block reuptake of neurotransmitters. Rationale CNS stimulation occurs when the amount of neurotransmitters being released and the duration of action of excitatory neurotransmitters are increased.

The nurse explains to a patient using caffeine that which disease process/condition may be exacerbated by this drug? A. Myelin degeneration B. Cardiac dysrhythmias C. Constipation D. Heart block

B. Cardiac dysrhythmias Rationale Caffeine stimulates the central nervous system, causing sympathomimetic effects, including cardiac dysrhythmias.

Which laboratory test should be monitored frequently to assess for a potential life-threatening adverse reaction to clozapine (Clozaril)? A. Renal panel B. Complete blood count C. Liver function tests D. Immunoglobulin levels

B. Complete blood count Rationale Patients taking clozapine must be monitored for the life-threatening side effect of agranulocytosis, evidenced by a severe reduction in the number of white blood cells.

2. The nurse will monitor for which adverse reaction to aluminum-containing antacids such as aluminum hydroxide (Amphojel)? A. Diarrhea B. Constipation C. Gastrointestinal upset D. Fluid retention

B. Constipation Rationale Aluminum- and calcium-containing antacids cause constipation, magnesium-containing antacids cause diarrhea, and sodium-containing antacids cause sodium and fluid retention.

. The nurse monitors a patient taking tolterodine (Detrol) for which therapeutic effect? A. Decrease in gastrointestinal motility B. Decrease in urinary frequency C. Increase in heart rate D. Increase in blood pressure

B. Decrease in urinary frequency Rationale Tolterodine (Detrol) blocks the muscarinic receptors in the bladder to decrease urinary frequency and urgency.

After administering oxybutynin (Ditropan) to a patient with spina bifida, the nurse is assessing the patient for therapeutic effects. What is the nurse assessing for in the patient? A. Decreased muscle twitches B. Decreased urinary frequency C. Increased bowel movements D. Increased heart rate

B. Decreased urinary frequency Rationale Oxybutynin (Ditropan) is a synthetic antimuscarinic drug used for the treatment of overactive bladder. It is also used as an antispasmodic for neurogenic bladder associated with spinal cord injuries and congenital conditions such as spina bifida.

When providing general education on use of over-the-counter medications for allergies, which is the highest priority for the nurse to include? A. Discontinue use 4 days before allergy testing. B. Do not use in patients younger than 2 years of age, unless prescribed. C. It may cause dry mouth. D. The medication treats the symptoms but is not a cure.

B. Do not use in patients younger than 2 years of age, unless prescribed. Rationale The Food and Drug Administration (FDA) recommends not using over-the-counter medications for allergies in children under the age of 2. A health care provider may prescribe the medications for this age group.

The addition of continuous administration of progestin to an estrogen regimen reduces the risk of which cancer? A. Ovarian cancer B. Endometrial cancer C. Breast cancer D. Vaginal cancer

B. Endometrial cancer Rationale Estrogen, given alone, has been associated with an increased risk of endometrial hyperplasia, which can lead to endometrial cancer. Progestin reduces the incidence of endometrial hyperplasia.

A new vaccination, Zostavax, has been approved to prevent the development of what condition in adults over the age of 50? A. Herpes simplex B. Herpes zoster C. Influenza A D. Avian influenza

B. Herpes zoster Rationale Zostavax is a new vaccination developed and approved for use in adults over the age of 50 to prevent the onset or outbreak of herpes zoster (shingles).

. Which vaccination was developed to prevent bacterial meningitis caused by Haemophilus influenzae? A. Hepatitis B vaccine B. Hib conjugate vaccine C. Prevnar D. Gardasil

B. Hib conjugate vaccine Rationale H. influenzae type b (Hib) (HibTITER, ActHIB, Liquid PedvaxHIB) vaccine is a noninfectious, bacteria-derived vaccine. Before this vaccine was developed, infections caused by Hib were the leading cause of bacterial meningitis in children 3 months to 5 years of age.

A female patient arrives in the clinic for counseling on potential hormone replacement therapy. When taking the patient history, which finding would the nurse consider as a contraindication to use of hormone replacement therapy for the patient? A. High cholesterol B. History of thromboembolic events C. Early menstrual onset D. High number of pregnancies

B. History of thromboembolic events Rationale A contraindication for estrogen administration is any history of thromboembolic disorder.

What would be a priority nursing diagnosis for a patient receiving anticholinergic (parasympatholytic) drugs? A. Risk for injury related to excessive central nervous system stimulation B. Impaired gas exchange related to thickened respiratory secretions C. Urinary retention related to loss of bladder tone D. Deficient knowledge related to pharmacologic regimen

B. Impaired gas exchange related to thickened respiratory secretions Rationale Although all of these nursing diagnoses are appropriate, the priority is determined remembering the ABCs. Anticholinergic drugs decrease respiratory secretions, which could lead to mucous plugs and resultant impaired gas exchange.

Cholinergic (parasympathomimetic) drugs have which therapeutic effect? A. Urinary retention B. Increased gastrointestinal motility C. Mydriasis D. Vasoconstriction

B. Increased gastrointestinal motility Rationale Cholinergic effects mimic the parasympathetic nervous system (rest and digest) as opposed to the sympathetic nervous system (fight or flight). Increasing gastrointestinal (GI) motility helps the body digest. Urinary retention, mydriasis, and vasoconstriction are sympathetic nervous system responses.

To reduce the gastrointestinal side effects of orlistat (Xenical), what will the nurse encourage the patient to do? A. Take the medication with an antacid. B. Limit dietary intake of fat to <30% of total calories. C. Supplement diet with fat-soluble vitamins. D. Increase fluid and fiber in the diet.

B. Limit dietary intake of fat to <30% of total calories. Rationale Orlistat is an anorexiant that works by blocking the absorption of fat from the gastrointestinal tract. Limiting intake of fat reduces the gastrointestinal side effects associated with increased fat content in stool (flatulence, oily spotting, and fecal urgency).

Which laboratory test would the nurse expect to be ordered to monitor a patient for adverse effects related to progestin medications? A. Cardiac enzymes B. Liver function tests C. BUN and creatinine D. Complete blood count

B. Liver function tests Rationale One of the most serious undesirable adverse effects of progestins is liver dysfunction, which would be assessed by monitoring liver enzymes.

Cholinergic (parasympathomimetic) drugs are indicated for which situation? A. Treating a postoperative patient who has bradycardia B. Lowering intraocular pressure in patients with glaucoma C. Inhibiting muscular activity in the bladder D. Preventing salivation and sweating

B. Lowering intraocular pressure in patients with glaucoma Rationale Cholinergic drugs stimulate the pupil to constrict (miosis), thus decreasing intraocular pressure.

2. When caring for a patient with serum potassium of 2.8 mEq/L, which is a priority nursing intervention when administering intravenous replacement therapy? A. Administer potassium as a bolus over 10 minutes. B. Maintain infusion rate at no greater than 20 mEq/hr. C. Apply ice packs to site of intravenous administration. D. Teach the patient and family the signs and symptoms of hypokalemia.

B. Maintain infusion rate at no greater than 20 mEq/hr. Too rapid an infusion of potassium can cause cardiac arrhythmias. Therefore intravenous potassium infusion rates should not exceed 20 mEq/hr.

The patient's chart notes the administration of dantrolene (Dantrium) immediately postoperatively. What does the nurse expect the patient has experienced? A. Delirium tremens B. Malignant hyperthermia C. Tonic-clonic seizure D. Respiratory arrest

B. Malignant hyperthermia Rationale Dantrolene is a direct-acting musculoskeletal muscle relaxant and is the drug of choice to treat malignant hyperthermia, a complication of generalized anesthesia.

What system assessment will the nurse monitor in a patient due to a known common side effect of traditional antihistamines? A. Cardiac status B. Neurologic status C. Respiratory status D. Gastrointestinal status

B. Neurologic status Rationale The most common side effect of antihistamines is sedation.

Which vaccination is marketed and recommended in the prevention of a virus that is known to cause cervical cancer? A. Herpes zoster vaccine (Zostavax) B. Papillomavirus vaccine (Gardasil) C. Pneumococcal vaccine (Prevnar 13) D. Hepatitis B virus vaccine (Recombivax HB)

B. Papillomavirus vaccine (Gardasil) Rationale Human papillomavirus virus (HPV) is a common cause of genital warts and cervical cancer. The papillomavirus vaccine (Gardasil, Cervarix) is the first and only vaccine known to prevent cancer.

8. The nurse is receiving shift report on her patients. One patient, per report, had a new IV bag of normal saline (NS) hung, which is to be infused at a rate of 100 mL per hour. The nurse discovers that the bag is almost empty and the pump was mistakenly set at 1000 mL per hour. What is the most immediate concern for any patient in this situation? A. Edema of hands and feet B. Pulmonary edema C. Increased urination D. Phlebitis

B. Pulmonary edema Crystalloids are a very safe and effective means of replacing needed fluid, when administered at the correct dosage. Because they contain no large particles, such as proteins, they do not stay within the blood vessels and can leak out of the plasma into the tissues and cells. This may result in edema anywhere in the body. Peripheral edema and pulmonary edema are two common examples. Pulmonary edema is the most concerning issue at this point due to the complications that can arise from fluid in the lungs. The nurse needs to stop the infusion and notify the provider immediately.

5. Which nursing diagnosis is appropriate for a patient receiving famotidine (Pepcid)? A. Risk for infection related to immunosuppression B. Risk for injury related to thrombocytopenia C. Impaired urinary elimination related to retention D. Ineffective peripheral tissue perfusion related to hypertension

B. Risk for injury related to thrombocytopenia Rationale A serious side effect of famotidine is thrombocytopenia, which is manifested by a decrease in platelet count and an increased risk of bleeding. The patient receiving famotidine may experience hypotension as an adverse effect, not hypertension. Famotidine does not cause immunosuppression or urinary retention.

A nurse working with patients who are diagnosed with ADHD is aware such patients often take CNS stimulant drugs. These medications are potent with a high potential for abuse and dependence. Based on this potential, how are these medications classified? A. Schedule I B. Schedule II C. Schedule III D. Schedule IV

B. Schedule II Rationale CNS stimulants are the first-line drugs of choice for both ADHD and narcolepsy. They are potent drugs with a strong potential for tolerance and psychological dependence and are therefore classified as Schedule II drugs under the Controlled Substances Act.

What is another approved and indicated use for bupropion (Zyban), a second-generation antidepressant? A. Orthostatic hypotension B. Smoking cessation C. Anorexia in patients undergoing chemotherapy D. Nocturnal enuresis in children

B. Smoking cessation Rationale Zyban is a sustained-release form of bupropion that is useful in helping patients quit smoking.

A patient currently prescribed duloxetine (Cymbalta) comes to the health clinic complaining of restlessness, agitation, diaphoresis, and tremors. The nurse suspects serotonin syndrome and questions the patient regarding concurrent use of which substance? A. ibuprofen B. St. John's wort C. vitamin E supplements D. glucosamine chondroitin

B. St. John's wort Rationale Serotonin syndrome may occur with SSRIs when they are combined with herbal products such as ginseng and St. John's wort.

When providing teaching to a patient diagnosed with myasthenia gravis, which instruction regarding the administration of physostigmine (Antilirium) is most appropriate? A. Increase fluid and fiber in the diet to prevent constipation. B. Take the medication 30 minutes before meals. C. If a dose is missed, double the next dose to prevent withdrawal. D. Common side effects include tachycardia and hypertension.

B. Take the medication 30 minutes before meals. Rationale Drugs used for myasthenia gravis should be given about 30 minutes before meals to allow for onset of action and therapeutic effects (e.g., decreased dysphagia). Constipation, tachycardia, and hypertension are not effects of cholinergic medications. A missed dose should never be doubled.

The nurse receives lab values for a patient with a theophylline level of 14 mcg/mL. How does the nurse interpret this theophylline level? A. Subtherapeutic B. Therapeutic C. Toxic D. Life-threatening

B. Therapeutic Rationale The therapeutic theopylline level is 10 to 20 mcg/mL.

How will the nurse describe the action of proton pump inhibitors? A. They compete with histamine for binding sites on the parietal cells. B. They irreversibly bind to H+/K+ATPase. C. They cause a decrease in stomach pH. D. They cannot be used to treat erosive esophagitis.

B. They irreversibly bind to H+/K+ATPase. Rationale Proton pump inhibitors work to block the final step in the acid-secreting mechanisms of the proton pump. They do this by irreversibly binding to H+/K+ ATPase, the enzyme for this step.

An elderly woman being treated for osteoporosis with a selective estrogen receptor modulator (SERM) should be alerted to the potential for an increased risk of which condition? A. Elevated blood pressure B. Venous thromboembolism C. Hypercalcemia D. Skin color changes

B. Venous thromboembolism Rationale Postmenopausal women taking SERMs are at an increased risk of development of thromboembolic events.

Discharge teaching for a patient receiving glucocorticoids would include the preferred use of which medication for pain management? A. aspirin B. acetaminophen C. ibuprofen D. naproxen

B. acetaminophen Rationale Acetaminophen does not cause gastrointestinal distress as does aspirin, ibuprofen, naproxen, and glucocorticoids.

A patient is admitted to the emergency department after taking an overdose of a barbiturate 15 minutes prior to arrival. The nurse can anticipate that which drug will be prescribed? A. naloxone (Narcan) B. activated charcoal C. flumazenil (Romazicon) D. ipecac syrup

B. activated charcoal Rationale There is no antidote for barbiturates. The use of activated charcoal absorbs any drug in the gastrointestinal tract, preventing absorption.

The nurse is obtaining a medication history from an 18-year-old female patient who has been diagnosed with genital herpes. Which drug would the nurse expect this patient to be prescribed? A. amantadine (Symmetrel) B. acyclovir (Zovirax) C. zidovudine (Retrovir) D. ribavirin (Virazole)

B. acyclovir (Zovirax) Rationale Acyclovir is the drug of choice to treat herpes simplex infections. Ribavirin is effective against respiratory syncytial virus (RSV), zidovudine against human immunodeficiency virus (HIV), and amantadine against Haemophilus influenzae type A.

A patient diagnosed with an anxiety disorder has been using lorazepam (Ativan) but finds the side effect of drowsiness to be interfering with life. Which anxiolytic medications might be a better option for this patient? A. alprazolam (Xanax) B. buspirone (BuSpar) C. chlordiazepoxide (Librium) D. hydroxyzine hydrochloride salt (Vistaril)

B. buspirone (BuSpar) Rationale Buspirone (BuSpar) is a third-generation anxiolytic that has the advantage of being both nonsedating and non-habit-forming. All the other options are sedatives as well as anxiolytics.

The dose-limiting toxicity of ganciclovir treatment is assessed by monitoring A. liver function tests (LFTs). B. complete blood count (CBC). C. blood urea nitrogen (BUN). D. creatine phosphokinase (CPK).

B. complete blood count (CBC). Rationale Bone marrow suppression is a dose-limiting toxicity of ganciclovir; therefore, CBC should be monitored.

The nurse administering donepezil (Aricept) to a patient understands that the expected therapeutic action of this drug is to A. relieve anxiety and restless behavior of the patient. B. increase levels of acetylcholine in the brain by blocking its breakdown. C. block the effects of acetylcholine at the presynaptic neurons. D. help control associated urinary incontinence.

B. increase levels of acetylcholine in the brain by blocking its breakdown. Rationale Donepezil is used to treat Alzheimer's disease, a disorder of decreased acetylcholine levels in the brain. Donepezil is an indirect-acting anticholinesterase drug.

Loratadine (Claritin) has an advantage over traditional antihistamines such as diphenhydramine (Benadryl) in that loratadine has A. less risk of cardiac arrhythmias. B. less sedative effect. C. increased bronchodilating effects. D. less gastrointestinal upset.

B. less sedative effect. Rationale Loratadine does not affect the central nervous system and therefore is nonsedating.

Propranolol (Inderal) is an effective A. selective alpha-adrenergic antagonist. B. nonselective beta-adrenergic antagonist. C. beta1-adrenergic antagonist. D. beta2-adrenergic antagonist.

B. nonselective beta-adrenergic antagonist. Rationale Propranolol is nonselective; it blocks both beta1 and beta2 receptors at therapeutic doses.

Which is the most common drug used for induction of labor in pregnant women and to promote ejection of milk during lactation? A. clomiphene B. oxytocin C. misoprostol D. mifepristone

B. oxytocin Rationale Oxytocin is a naturally occurring hormone that can be given in synthetic form for induction of labor.

During a postpartum assessment, the nurse notes a boggy uterus and increased vaginal bleeding. Based upon this assessment and standing physician orders, the nurse prepares to administer which medication? A. prostaglandin E B. oxytocin (Pitocin) C. terbutaline (Brethine) D. clomiphene (Clomid)

B. oxytocin (Pitocin) Rationale Oxytocin is a uterine stimulant that causes uterine contractions, which would decrease the vaginal bleeding.

5. During assessment of a patient diagnosed with pheochromocytoma, the nurse auscultates a blood pressure of 210/110 mm Hg. The nurse would expect to administer which medication? A. nadolol (Corgard) B. phentolamine (Regitine) C. dobutamine (Dobutrex) D. verapamil (Calan)

B. phentolamine (Regitine) Rationale Phentolamine is a potent alpha-blocking drug specifically effective for treatment of hypertension associated with pheochromocytoma.

The nurse assesses the intravenous infusion site of a patient receiving dopamine and finds it is infiltrated. The nurse will prepare which medication to treat this infiltration? A. lidocaine (Xylocaine) B. phentolamine (Regitine) C. nitroprusside (Nipride) D. naloxone (Narcan)

B. phentolamine (Regitine) Rationale Phentolamine is an alpha blocker that causes vasodilation, thus counteracting the vasoconstrictive effects of the infiltrated dopamine. The vasodilation will increase blood flow to the site and decrease the risk of tissue necrosis.

A patient complains of worsening nasal congestion despite the use of oxymetazoline (Afrin) nasal spray every 2 to 4 hours for the past 5 days. The nurse's response is based on knowledge that A. oxymetazoline is not an effective nasal decongestant. B. sustained use of nasal decongestants over several days may result in rebound congestion. C. oxymetazoline should be administered in an hourly regimen for severe congestion. D. the patient is probably displaying an idiosyncratic reaction to oxymetazoline.

B. sustained use of nasal decongestants over several days may result in rebound congestion. Rationale Oxymetazoline is an effective nasal decongestant, but overuse results in worsening or "rebound" congestion. It should not be used more than every 4 hours.

During postoperative teaching, the nurse explains that the patient is receiving bethanechol (Urecholine) to treat A. postoperative hypotension. B. urinary atony. C. respiratory atelectasis. D. postoperative ischemic colitis.

B. urinary atony. Rationale Bethanechol is a direct-acting cholinergic agonist that stimulates the cholinergic receptors on the smooth muscle of the bladder, leading to bladder contraction and emptying.

Which statement, if made by a patient with HIV infection, demonstrates a need for continued patient teaching? A. "I will change my position slowly to prevent dizziness and potential injury." B. "I must take these medications exactly as prescribed for the rest of my life." C. "I don't need to use condoms as long as I take my medication as prescribed." D. "I should remain upright for 30 minutes after taking my zidovudine."

C. "I don't need to use condoms as long as I take my medication as prescribed." Rationale Antiretroviral drugs do not stop the transmission of HIV, and patients need to continue standard precautions.

Which statement, made by the patient, indicates an understanding of discharge teaching regarding alendronate (Fosamax)? A. "I need to decrease my intake of dairy products so as to prevent hypercalcemia." B. "I need to take this medication with food to prevent damage to my esophagus." C. "I will take the medication first thing in the morning with 8 ounces of water and remain upright for 30 minutes." D. "This medication will help relieve the bone pain I have from my osteoporosis."

C. "I will take the medication first thing in the morning with 8 ounces of water and remain upright for 30 minutes." Rationale Alendronate can cause erosive esophagitis. To prevent this side effect, it is important to take the medication first thing in the morning on an empty stomach without any other medications and maintain an upright position for 30 minutes. These actions facilitate rapid absorption and prevent reflux into the esophagus.

Which statement made by a patient demonstrates a lack of understanding of patient teaching regarding phenothiazine drug therapy? A. "I need to change positions slowly to prevent dizziness." B. "I will need to wear sunscreen and protective clothing when outdoors." C. "It is okay to take this drug with a small glass of wine to help relax me." D. "I should call my doctor if I notice any uncontrollable movements of my tongue."

C. "It is okay to take this drug with a small glass of wine to help relax me." Rationale Drinking alcohol with phenothiazines puts the patient at risk for increased central nervous system depression.

During patient teaching, the nurse explains the difference between a sedative and hypnotic with which statement? A. "Sedatives are much stronger than hypnotic drugs and should only be used for short periods of time." B. "Sedative drugs induce sleep, whereas hypnotic drugs induce a state of hypnosis." C. "Most drugs produce sedation at low doses and sleep, the hypnotic effect, at higher doses." D. "There really is no difference; the terms are used interchangeably."

C. "Most drugs produce sedation at low doses and sleep, the hypnotic effect, at higher doses." Rationale Many drugs have both sedative and hypnotic properties, with the sedative properties evident at low doses and the hypnotic properties demonstrated at larger doses.

When educating a patient about the use of oral contraceptives, the nurse provides what explanation for the pills having different colors each week? A. "They help you remember which week you are taking." B. "There isn't a reason for the color, just the choice of the drug company." C. "The different color pills are due to different amounts of hormones in each week." D. "They are color coded for the weeks of the month."

C. "The different color pills are due to different amounts of hormones in each week." Rationale Oral contraceptive pills can be monophasic, biphasic, or triphasic. If the pills are different colors, they have different amounts of hormones and are likely the triphasic type of contraceptive.

Patient teaching regarding the use of antileukotriene drugs such as zafirlukast (Accolate) would include which statement? A. "Take the medication as soon as you begin wheezing." B. "It will take about 3 weeks before you notice a therapeutic effect." C. "This medication works by preventing the inflammation that causes your asthma attack." D. "Increase fiber and fluid in your diet to prevent the common side effect of constipation."

C. "This medication works by preventing the inflammation that causes your asthma attack." Rationale Antileukotriene drugs block the inflammatory response of leukotrienes and thus the trigger for asthma attacks. Response to these drugs is usually noticed within 1 week. They are not used to treat an acute asthma attack. Diarrhea, not constipation, is an adverse effect of the antileukotriene drugs.

Which statement would be included when teaching a patient about the proper use of metered-dose inhalers? A. "After you inhale the medication once, repeat until you obtain relief." B. "Make sure that you puff out air repeatedly after you inhale the medication." C. "Wait 1 to 2 minutes before you take a second puff of the same drug." D. "Hold the inhaler in your mouth, take a deep breath, and then compress the inhaler."

C. "Wait 1 to 2 minutes before you take a second puff of the same drug." Rationale If a second puff of the same drug is ordered, instruct the patient to wait 1 to 2 minutes between puffs. If a second type of inhaled drug is ordered, instruct the patient to wait 2 to 5 minutes between the medications or to take as prescribed.

An adult patient presents to the emergency room with insecticide poisoning. The nurse prepares to administer which dose of atropine intravenously? A. 0.3 mg B. 0.5 mg C. 2 mg D. 4 mg

C. 2 mg Rationale Higher doses of atropine (1 to 3 mg) are needed to treat cholinergic overdoses such as insecticide poisoning.

6. Which would be the most appropriate application time for a patient prescribed a scopolamine patch for motion sickness? A. At bedtime B. Every 4 hours as needed C. 4 to 5 hours before travel D. At the first sign of motion sickness

C. 4 to 5 hours before travel Rationale For the prevention of motion sickness, scopolamine is available in a convenient transdermal delivery system (a patch) that can be applied just behind the ear 4 to 5 hours before travel. Each patch has a 72-hour duration of action, thus necessitating a change only every 3 days.

6. The nurse is assessing a patient noted to have third spacing and edema of the hands and feet at +3, yet the patient is having signs and symptoms of intravascular dehydration. The patient's lab results report a total protein level of 4.6 g/dL. What fluid does the nurse anticipate the provider will order for this patient? A. Normal saline B. Lactated Ringer's C. 5% albumin D. Whole blood

C. 5% albumin If the total protein level falls below 5.3 g/dL, fluid shifts out of blood vessels into the tissues. When this happens, colloid replacement therapy is required to reverse this process by increasing the colloid oncotic pressure. The three most commonly used are 5% albumin, dextran 40, and hetastarch. They all have a very rapid onset of action as well as a long duration of action.

The nurse will question an order for misoprostol (Cytotec) in which patient? A. A 21-year-old man with Zollinger-Ellison syndrome B. A 64-year-old man with hypertension C. A 32-year-old pregnant woman with a urinary tract infection D. A 45-year-old woman with GERD

C. A 32-year-old pregnant woman with a urinary tract infection Rationale Misoprostol (Cytotec) is a prostaglandin E analogue and is believed to inhibit gastric acid secretion and protect the gastric mucosa from injury by enhancing the local production of mucus. However, it is also an abortifacient and therefore would be contraindicated in pregnancy. The drug may be useful in treating patients with Zollinger-Ellison syndrome (a hypersecretory syndrome) and GERD. Hypertension is not a contraindication for its use.

Proton pump inhibitors have the ability to almost totally inhibit gastric acid secretion. Due to this possibility, the use of the medication can lead to what problem? A. Gastric ulcer formation B. Gastroesophageal reflux disease (GERD) C. Achlorhydria D. Diverticulosis

C. Achlorhydria Rationale Because proton pump inhibitors stop the final step of acid secretion, they can block up to 90% of acid secretion, leading to achlorhydria (without acid).

Mast cell stabilizers are most effective in treating bronchoconstriction associated with which condition? A. Emphysema B. Exposure to cold C. Allergens D. Infection

C. Allergens Rationale Mast cell stabilizers work by preventing the release of chemical mediators that cause bronchospasm and are most effective in preventing asthma caused by extrinsic factors such as allergens or exercise.

Which condition is not an anticipated side effect of azathioprine (Imuran)? A. Leukopenia B. Thrombocytopenia C. Alopecia D. Hepatotoxicity

C. Alopecia Rationale Common side effects of azathioprine include leukopenia, thrombocytopenia, and hepatotoxicity. Alopecia (hair loss) is not an expected side effect.

The provider has ordered donepezil (Aricept) for the patient, and the patient states "I have no idea why I take this medication." What is the most common diagnosis associated with the administration of donepezil (Aricept)? A. Parkinson's disease B. Bladder retention C. Alzheimer's disease D. Urinary retention

C. Alzheimer's disease Rationale Donepezil (Aricept) is a cholinesterase inhibitor drug that works centrally in the brain to increase levels of acetylcholine by inhibiting acetylcholinesterase. It is used in the treatment of mild to moderate Alzheimer's disease.

What is the priority nursing assessment to monitor when administering vaccinations? A. Pain at the injection site B. Signs and symptoms of infection C. Anaphylaxis D. Myalgias

C. Anaphylaxis Rationale Anaphylaxis is a potential life-threatening adverse reaction to vaccines. Pain and myalgias can occur but are not life threatening.

The nurse would question an order for pseudoephedrine in a patient with a history of which condition? A. Pneumonia B. Osteoporosis C. Atrial fibrillation D. Peptic ulcer disease

C. Atrial fibrillation Rationale Adrenergic drugs are usually contraindicated in patients with narrow-angle glaucoma, diabetes, cardiovascular disease, hyperthyroidism, prostatitis, or a known hypersensitivity to such drugs.

The nurse notes lithium on a patient's drug history upon admission. Which condition would the nurse suspect that this patient has been diagnosed with? A. Obsessive-compulsive disorder B. Absence seizures C. Bipolar disorder D. Paranoid schizophrenia

C. Bipolar disorder Rationale Lithium is an antimanic drug used to treat manic episodes associated with bipolar disorders.

How does sucralfate (Carafate) achieve a therapeutic effect? A. By neutralizing gastric acid B. By enhancing gastric absorption C. By forming a protective barrier over the gastric mucosa D. By inhibiting gastric acid secretion

C. By forming a protective barrier over the gastric mucosa Rationale Sucralfate has a local effect only on the gastric mucosa. It forms a pastelike substance in the stomach, which adheres to the gastric lining, protecting against adverse effects related to gastric acid. It also stimulates healing of any ulcerated areas of the gastric mucosa.

The nurse would teach a patient prescribed simethicone (Gas-X) to avoid which substances? A. Antacids B. Dairy products C. Carbonated beverages D. Histamine2-receptor antagonists

C. Carbonated beverages Rationale Simethicone is used to decrease gas and belching, both of which can be aggravated or caused by ingesting carbonated beverages. It may be given in combination with other medications used to decrease acidity.

The drug mycophenolate (CellCept) has a black box warning because of which potential adverse effect? A. Hypertension B. Suicidal ideations C. Congenital malformations D. Abnormal high number of adverse effects

C. Congenital malformations Rationale Mycophenolate (CellCept) is an antimetabolite and suppresses T cell proliferation. It is indicated for the prevention of organ rejection as well as the treatment of organ rejection. Mycophenolate has a black box warning from the Food and Drug Administration stating that it is associated with an increased risk of congenital malformations and spontaneous abortions when used during pregnancy.

4. A patient receiving a unit of red blood cells suddenly develops shortness of breath, chills, and fever. Following patient assessment, what is the nurse's initial action? A. Reassure the patient that this is an expected reaction. B. Notify the physician while a peer monitors the blood transfusion. C. Discontinue the infusion of packed cells. D. Decrease the infusion rate, and reassess the patient in 15 minutes.

C. Discontinue the infusion of packed cells. These are signs and symptoms of a blood transfusion reaction that could escalate to anaphylaxis. Therefore it is a priority to immediately stop the blood transfusion.

Which activity should the patient be cautioned to avoid while taking an MAO inhibitor? A. Participating in a bowling league B. Sunbathing at the pool C. Eating aged cheese D. Smoking a low-nicotine cigarette

C. Eating aged cheese Rationale Eating foods high in tyramine, including aged cheese, can cause a hypertensive crisis in patients taking MAO inhibitors.

The nurse would teach a patient prescribed cyclosporine to avoid which substance? A. Sunscreen B. Chocolate milk C. Grapefruit juice D. Acetaminophen

C. Grapefruit juice Rationale Patients should avoid consuming grapefruit or grapefruit juice because they will increase the blood concentrations of cyclosporine. Sunscreen should be used to avoid photosensitivity, and the medication should be taken with food or chocolate milk to prevent gastrointestinal upset.

When assessing for potential side effects of fludrocortisone (Florinef), the nurse monitors for signs and symptoms of which adverse effect? A. Hyponatremia B. Hypercalcemia C. Hypokalemia D. Hypovolemia

C. Hypokalemia Rationale Fludrocortisone has mineralocorticoid properties, resulting in sodium and fluid retention along with potassium excretion.

Patient teaching regarding expectorants should instruct the patient to perform which action? A. Restrict fluids to decrease mucus production. B. Take the medication once a day only, usually at bedtime. C. Increase fluid intake to decrease viscosity of secretions. D. Increase fiber and fluid intake to prevent constipation.

C. Increase fluid intake to decrease viscosity of secretions. Rationale Expectorant drugs are used to decrease viscosity of secretions and allow them to be more easily expectorated. Increasing fluid intake helps this action.

1. Epinephrine, as an adrenergic (sympathomimetic) drug, produces which therapeutic effect? A. Urinary retention B. Bronchial constriction C. Increased heart rate and contractility D. Decreased intestinal motility

C. Increased heart rate and contractility Rationale Epinephrine causes sympathomimetic actions, including increased heart rate and contractility. The other effects listed are parasympathomimetic in nature.

The nurse is providing education to a patient with a history of chronic nasal congestion secondary to allergic rhinitis. Which class of medications would the nurse anticipate the provider would recommend for the patient to use on a long-term basis? A. Anititussives B. Antihistamines C. Intranasal steroids D. Expectorants

C. Intranasal steroids Rationale Inhaled intranasal steroids and anticholinergic drugs are not associated with rebound congestion and are often used prophylactically to prevent nasal congestion in patients with chronic upper respiratory tract symptoms. Local intranasal steroids would have the least likely possible systemic side effects of all the medication classes possible to use for chronic and long term use.

The most significant drug interactions with use of antivirals occur when antivirals are administered via which route? A. Topically B. Rectally C. Intravenously D. Optically

C. Intravenously Rationale Significant drug interactions that occur with antiviral drugs arise most often when they are administered via systemic routes such as intravenously and orally. Many of these drugs are also applied topically to the eye or body, however, and the incidence of drug interactions associated with these routes of administration is much lower.

The nurse is discussing with a patient the time of day for taking prednisone. What information would the nurse include in the teaching based on knowledge of glucocorticoids? A. It is usually administered early in the evening to coincide with the natural secretion pattern of the adrenal cortex. B. It is usually administered on a strict, unchanging schedule in order to prevent adverse reactions. C. It should be administered with food to diminish the risk of gastric irritation. D. It should be administered with the patient's morning coffee to enhance its effects.

C. It should be administered with food to diminish the risk of gastric irritation. Rationale Glucocorticoids can cause gastrointestinal distress and should be administered with food. The normal circadian secretion of the adrenal cortex is early morning to wake the person up, not early evening. These medications should be tapered off slowly to prevent adrenal crisis and can be administered intravenously. While glucocorticoids should be given in the morning, they should not be administered with coffee, which contains caffeine and may increase gastric irritation.

3. Which type of antacids will the nurse most likely question in an order for a patient with chronic renal failure? A. Aluminum-containing antacids B. Calcium-containing antacids C. Magnesium-containing antacids D. Sodium-containing antacids

C. Magnesium-containing antacids Rationale Magnesium-containing antacids can cause hypermagnesemia in patients with chronic renal failure. Aluminum-containing antacids may be used as a phosphate binder in patients with chronic renal failure. Sodium- and aluminum-containing antacids are chemically more easily excreted in patients with renal compromise. Although calcium-containing antacids may accumulate in the bloodstream of patients with renal failure, they may also be appropriate because these patients may be hypocalcemic.

The patient is being treated with the drug fingolimod (Gilenya). What disease or disorder would the nurse expect the patient to have? A. Guillain-Barré syndrome B. Amyotrophic lateral sclerosis C. Multiple sclerosis D. Parkinson's disease and dementia

C. Multiple sclerosis Rationale A new drug, fingolimod (Gilenya), which actually failed as an antirejection drug, was approved in 2010 for multiple sclerosis. It is the only oral drug for relapsing forms of multiple sclerosis. It has significant adverse effects, including headache, hepatotoxicity, flulike symptoms, back pain, atrioventricular block, bradycardia, hypertension, and macular edema.

7. The nurse is providing care to a patient newly admitted for trauma from a motor vehicle accident. The provider has ordered IV fluids with potassium. What information noted in the patient's history would alert the nurse to question the order for this IV fluid? A. Hypertension B. Cirrhosis of the liver C. Renal failure D. Multiple sclerosis

C. Renal failure Potassium supplements are administered either to prevent or to treat potassium depletion. Potassium is contraindicated in patients with severe renal disease, severe hemolytic disease, or Addison's disease and in those with hyperkalemia, acute dehydration, or extensive tissue breakdown stemming from multiple traumas. The other history information is not contraindicated in the administration of potassium.

Which over-the-counter/herbal product, when taken with theophylline, can decrease theophylline's serum drug levels? A. Caffeine B. Diphenhydramine C. St. John's wort D. Echinacea

C. St. John's wort Rationale St. John's wort has been shown to enhance the rate of theophylline metabolism, thus decreasing serum levels.

What is the general action of immunosuppressants? A. They increase antibody response. B. They increase natural killer (NK) cellular activity. C. They suppress T-lymphocytes. D. They suppress hepatic metabolism of steroids.

C. They suppress T-lymphocytes. Rationale Immunosuppressants inhibit T-lymphocyte synthesis, thus preventing an immune response to organ transplants.

Medications used to treat HIV infections are more specifically classified as A. antifungal drugs. B. antiviral drugs. C. antiretroviral drugs. D. antiparasitic drugs.

C. antiretroviral drugs. Rationale HIV is a member of the retrovirus family; therefore drugs used to treat this virus are classified as antiretroviral drugs. Although antiretroviral drugs also fall under the broader category of antiviral drugs in general, their mechanisms of action are unique to the AIDS virus. So, they are more commonly referred to by their subclassification as antiretroviral drugs.

An example of a cardioselective beta blocker includes A. propranolol (Inderal). B. labetalol (Normodyne). C. atenolol (Tenormin). D. sotalol (Betapace).

C. atenolol (Tenormin). Rationale At therapeutic dosages, atenolol selectively blocks only the beta1 receptors in the heart, not the beta2 receptors located in the lungs.

The nurse is discussing use of antihistamines for allergic rhinitis. Which medications would be included in the list of H1 antagonists used in the treatment of allergic rhinitis? (Select all that apply.) A. ranitidine (Zantac) B. nizatidine (Axid) C. fexofenadine (Allegra) D. loratadine (Claritin) E. cetirizine (Zyrtec)

C. fexofenadine (Allegra) D. loratadine (Claritin) E. cetirizine (Zyrtec) Rationale Antihistamines are drugs that directly compete with histamine for specific receptor sites. For this reason, they are also called histamine antagonists. H1 antagonists include drugs such as diphenhydramine (Benadryl), chlorpheniramine (generic), fexofenadine (Allegra), loratadine (Claritin), and cetirizine (Zyrtec).

3. When planning administration of hypertonic saline solution to treat a patient with severe hyponatremia, the nurse monitors for signs and symptoms of overdose as manifested by A. lethargy and hypotension. B. vomiting and diarrhea. C. flushed skin and increased thirst. D. confusion and seizures.

C. flushed skin and increased thirst. Flushed skin and increased thirst are signs and symptoms of hypernatremia. The other choices are signs and symptoms of hyponatremia.

1. A patient with severe trauma is admitted to the intensive care unit. The patient has received 5000 mL of normal saline, is exhibiting peripheral edema, and remains hypotensive. The nurse anticipates administering which substance to correct fluid balance? A. Ringer's lactate B. A 3% saline solution C. hetastarch (Hespan) D. D5W

C. hetastarch (Hespan) The patient needs to increase intravascular fluid volume. Hetastarch will enable this since it is a colloid that will increase osmotic pull from the extravascular spaces to the intravascular area. A 3% saline solution is also hypertonic, but its use is not preferred secondary to risk of hypernatremia.

The nurse would anticipate administering which medication to a patient demonstrating acute organ rejection? A. azathioprine (Imuran) B. basiliximab (Simulect) C. muromonab-CD3 (Orthoclone OKT3) D. sirolimus (Rapamune)

C. muromonab-CD3 (Orthoclone OKT3) Rationale Only muromonab-CD3 (Orthoclone OKT3) is used to treat acute organ rejection. The other immunosuppressants are used to prevent organ rejection.

A patient using Afrin nasal spray complains of worsening cold symptoms and tells the nurse, "I don't understand why this is not working. I am using it almost every 3 hours!" The nurse's response is based on knowledge that A. Afrin nasal spray is not an effective nasal decongestant. B. the medication needs to be used every 30 minutes for maximum effectiveness. C. the patient is suffering from rebound congestion related to excessive use of the Afrin nasal spray. D. adrenergic decongestants should only be used prophylactically, not to treat acute congestion.

C. the patient is suffering from rebound congestion related to excessive use of the Afrin nasal spray. Rationale Afrin nasal spray is a sympathomimetic drug with both alpha- and beta-adrenergic effects. The alpha-adrenergic activity is responsible for causing vasoconstriction in the nasal mucosa. However, excessive use of nasal decongestants can lead to greater congestion because of a rebound phenomenon that occurs when use of the product is stopped.

Chapter 12 Central Nervous System Depressants and Muscle Relaxants

Chapter 12 Central Nervous System Depressants and Muscle Relaxants

Chapter 13 Central Nervous System Stimulants and Related Drugs

Chapter 13 Central Nervous System Stimulants and Related Drugs

Chapter 16 Psychotherapeutic Drugs

Chapter 16 Psychotherapeutic Drugs

Chapter 18 Adrenergic Drugs

Chapter 18 Adrenergic Drugs

Chapter 19 Adrenergic-Blocking Drugs

Chapter 19 Adrenergic-Blocking Drugs

Chapter 20 Cholinergic Drugs

Chapter 20 Cholinergic Drugs

Chapter 21 Cholinergic-Blocking Drugs

Chapter 21 Cholinergic-Blocking Drugs

Chapter 29 Fluids and Electrolytes

Chapter 29 Fluids and Electrolytes

CHAPTERS FROM PHARMACOLOGY & THE NURSING PROCESS, 7TH EDITION

Chapter 29 Fluids and Electrolytes Chapter 50 Acid-Controlling Drugs Chapter 18 Adrenergic Drugs Chapter 19 Adrenergic-Blocking Drugs Chapter 20 Cholinergic Drugs Chapter 21 Cholinergic-Blocking Drugs Chapter 33 Adrenal Drugs Chapter 40 Antiviral Drugs Chapter 49 Immunizing Drugs and Biochemical Terrorism Chapter 12 Central Nervous System Depressants and Muscle Relaxants Chapter 13 Central Nervous System Stimulants and Related Drugs Chapter 48 Immunosuppressant Drugs Chapter 16 Psychotherapeutic Drugs Chapter 34 Women's Health Drugs Chapter 36 Antihistamines, Decongestants, Antitussives, and Expectorants Chapter 37 Respiratory Drugs

Chapter 33 Adrenal Drugs

Chapter 33 Adrenal Drugs

Chapter 34 Women's Health Drugs

Chapter 34 Women's Health Drugs

Chapter 36 Antihistamines, Decongestants, Antitussives, and Expectorants

Chapter 36 Antihistamines, Decongestants, Antitussives, and Expectorants

Chapter 37 Respiratory Drugs

Chapter 37 Respiratory Drugs

Chapter 40 Antiviral Drugs

Chapter 40 Antiviral Drugs

Chapter 48 Immunosuppressant Drugs

Chapter 48 Immunosuppressant Drugs

Chapter 49 Immunizing Drugs and Biochemical Terrorism

Chapter 49 Immunizing Drugs and Biochemical Terrorism

Chapter 50 Acid-Controlling Drugs

Chapter 50 Acid-Controlling Drugs

What statement by the patient, who is using the anticholinergic inhaler ipratropium bromide (Atrovent), indicates to the nurse that teaching has been successful? A. "I will increase my intake of caffeine." B. "I may gain weight as a result of taking this medication." C. "Nausea and vomiting are common adverse effects of this medication, so I will always take it with meals." D. "This inhaler is not to be used alone to treat an acute asthma attack."

D. "This inhaler is not to be used alone to treat an acute asthma attack." Rationale Although ipratropium works to prevent bronchoconstriction and thus secondarily leads to bronchodilation, a direct-acting bronchodilator is needed to treat an acute asthma attack.

The nurse would question an order for aminoglutethimide in a patient with which condition? A. Metastatic breast cancer B. Cushing's syndrome C. Adrenal malignancy D. Addison's disease

D. Addison's disease Rationale Aminoglutethimide suppresses the adrenal cortex. Addison's disease presents with decreased adrenal secretion; thus you would not want to exacerbate this by administering aminoglutethimide. All of the other choices are indications for use for aminoglutethimide.

An allergy to which substance is listed as a contraindication to the administration of an immunizing drug? A. Soy B. Corn C. Wheat D. Egg

D. Egg Rationale Contraindications to the administration of immunizing drugs include allergy to the immunization itself or allergy to any of its components, such as eggs or yeast.

Which is an adverse effect of bethanechol (Urecholine)? A. Constipation B. Hypertension C. Tachycardia D. Headache

D. Headache Rationale Adverse effects of bethanechol include abdominal cramps, diarrhea, hypotension, bradycardia, and headache.

Administration of which of the following provides passive immunity? A. Vaccines B. Toxoids C. Antitoxins D. Immunoglobulins

D. Immunoglobulins Rationale Vaccines, antitoxins, and toxoids provide active immunity by stimulating the humoral immune system. Immunoglobulins provide passive immunity by giving the patient substances to fight specific antigens.

A nurse is providing education to a patient taking two different medications. The nurse identifies which characteristic as the advantage of salmeterol (Serevent) over other beta2 agonists such as albuterol (Proventil)? A. Shorter onset of action B. Better side effect profile C. Quicker peak action D. Longer duration of action

D. Longer duration of action Rationale Salmeterol has a longer duration of action, requiring the patient to use it only twice a day instead of four times a day with albuterol.

A patient diagnosed with depression is started on a TCA after failure to improve symptoms on an SSRI. The nurse should include which teaching point when educating the patient about the new medication? A. There are no contraindications to this medication. B. The medication is safe; it has been used longer than many others. C. This class of medications has no other use and is only for depression. D. There is a risk of toxicity when this medication is taken with alcohol.

D. There is a risk of toxicity when this medication is taken with alcohol. Rationale There is an increased risk of toxicity with TCAs when taken with alcohol and a high rate of morbidity.

What is the role of corticosteroids in the treatment of acute respiratory disorders? A. They stimulate the immune system. B. They directly dilate the bronchi. C. They increase gas exchange in the alveoli. D. They decrease inflammation.

D. They decrease inflammation. Rationale Corticosteroids suppress the immune system. They do not directly affect bronchodilation but rather prevent bronchoconstriction as a response to inflammation.

How is the effectiveness of antiviral drugs administered to treat HIV infection assessed and evaluated? A. Megakaryocytes B. Red blood cell counts C. Lymphocyte counts D. Viral load

D. Viral load Rationale All antiretroviral drugs work to reduce the viral load, which is the number of viral RNA copies per milliliter of blood.

When assessing for cardiovascular effects of a beta blocker, the nurse understands that these drugs produce A. a positive inotropic, positive chronotropic, and positive dromotropic effect. B. a positive inotropic, negative chronotropic, and negative dromotropic effect. C. a negative inotropic, positive chronotropic, and positive dromotropic effect. D. a negative inotropic, negative chronotropic, and negative dromotropic effect.

D. a negative inotropic, negative chronotropic, and negative dromotropic effect. Rationale Beta blockers exert a sympatholytic effect, blocking the effects of sympathetic nervous system stimulation, causing a decrease in heart rate (negative chronotropic), conductivity (negative dromotropic), and contractility (negative inotropic).

When teaching a patient about beta blockers such as atenolol (Tenormin) and metoprolol (Lopressor), it is important to inform the patient that A. these medications may be taken with antacids to minimize gastrointestinal distress. B. hot baths and showers will help enhance the therapeutic effects and are encouraged. C. alcohol intake is encouraged for its vasodilating effects. D. abrupt medication withdrawal may lead to a rebound hypertensive crisis.

D. abrupt medication withdrawal may lead to a rebound hypertensive crisis. Rationale Abrupt withdrawal of a beta-blocking drug can cause rebound hypertension. These drugs should be gradually decreased.

A patient is admitted to the emergency department with a severe overdose of a benzodiazepine. The nurse immediately prepares to administer which antidote from the emergency drug cart? A. naloxone (Narcan) B. naltrexone (ReVia) C. nalmefene (Revex) D. flumazenil (Romazicon)

D. flumazenil (Romazicon) Rationale Flumazenil is the antidote for benzodiazepine overdose. The other options are only effective against opioid effects.

A hypertensive crisis may occur if adrenergic (sympathomimetic) drugs are given along with A. beta blockers. B. diuretics. C. alpha1 blockers. D. monoamine oxidase (MAO) inhibitors.

D. monoamine oxidase (MAO) inhibitors. Rationale Adrenergic drugs combined with MAO inhibitors can lead to extreme hypertension. All the other drugs listed are used to treat hypertension.

What atypical antipsychotic medication would the nurse anticipate a provider prescribing for treatment of refractory schizophrenia? A. trazodone (Desyrel) B. phenelzine (Nardil) C. amoxapine (Asendin) D. risperidone (Risperdal)

D. risperidone (Risperdal) Rationale Risperidone is effective for refractory schizophrenia, including negative symptoms. The other medications listed are antidepressants.

Nonselective beta blockers may be used to treat hypertension and A. chronic obstructive pulmonary disease (COPD). B. heart failure. C. heart block. D. supraventricular dysrhythmias.

D. supraventricular dysrhythmias. Rationale Nonselective beta blockers are used to treat supraventricular dysrhythmias secondary to their negative chronotropic effects (decreasing heart rate). They may exacerbate heart failure, COPD, and heart block secondary to their negative inotropic effect (heart failure), blocking of bronchodilation (COPD), and negative chronotropic effects (heart block).


संबंधित स्टडी सेट्स

Chapter 16: Injuries to the Lower Leg, Ankle And Foot

View Set

new and improved world history study guide lol

View Set

The flow of food: Service Chp. 7

View Set

public and other goods; international trade

View Set

3.3 Some Proteins Act as Enzymes to Speed up Biochemical Reactions

View Set

Pearson Chapter 19 Review Questions

View Set